The question bank may take some time to load… Just enough time to stretch, blink a few times, and question your life choices — but not too long, we promise!
We recommend going Full Screen for the best experience. Have Fun !
Report a question
Endo – 2017
Questions from The 2017 Module + Annual Exam of Endocrinology
Consider which hormone plays a crucial role in both childbirth and breastfeeding . Then think about the hypothalamic nucleus known for initiating those life-sustaining contractions and reflexes.
1 / 109
Category:
Endo – Physio
Which of the following is secreted by the paraventricular nucleus?
The paraventricular nucleus (PVN) is a key part of the hypothalamus and has both neuroendocrine and autonomic functions.
The PVN primarily synthesizes:
Oxytocin , which:
Causes uterine contractions during labor
Triggers the milk ejection reflex during breastfeeding
Also plays a role in emotional bonding and social behavior
It also produces:
After synthesis, these hormones travel via axons down the hypothalamo-hypophyseal tract to be stored and released from the posterior pituitary .
So, while both oxytocin and some ADH are made in the PVN, oxytocin is more definitively and classically associated with this nucleus .
Incorrect Answer Explanations:
Antidiuretic hormone (ADH)
Gonadotropin-releasing hormone (GnRH)
Incorrect
GnRH is produced in the preoptic area of the hypothalamus, not in the paraventricular nucleus.
It controls the release of LH and FSH from the anterior pituitary.
Prolactin
Incorrect
Prolactin is secreted by the anterior pituitary , not the hypothalamus.
Its secretion is primarily regulated (inhibited) by dopamine from the hypothalamus.
Adrenocorticotropic hormone (ACTH)
Incorrect
ACTH is produced by the anterior pituitary , stimulated by CRH (corticotropin-releasing hormone) from the hypothalamus.
It is not secreted by the paraventricular nucleus.
When a hormone causes the kidneys to retain sodium and excrete potassium and hydrogen ions , what happens to blood pH? Consider the systemic effect of prolonged hydrogen ion loss and how that shifts acid–base balance.
2 / 109
Category:
Endo – Pathology
Which metabolic derangement is seen in Conn syndrome?
Conn syndrome , also known as primary hyperaldosteronism , is caused by excess secretion of aldosterone , typically due to an adrenal adenoma or adrenal hyperplasia.
Physiological Role of Aldosterone:
Aldosterone acts on the distal nephron (especially the collecting ducts) of the kidneys to:
Increase sodium reabsorption (→ water retention → hypertension)
Increase potassium excretion (→ hypokalemia)
Increase hydrogen ion excretion (→ metabolic alkalosis)
Why Metabolic Alkalosis?
Incorrect Answer Explanations:
Hyperkalemia
Hypercalcemia
Metabolic acidosis
Incorrect : This would occur if hydrogen ions were retained , such as in renal failure or type 1 renal tubular acidosis .
In Conn syndrome, H⁺ is excessively excreted , leading to alkalosis.
Respiratory acidosis
Incorrect : This is due to hypoventilation , causing CO₂ retention (e.g., in COPD or CNS depression).
Conn syndrome is a metabolic , not respiratory, disorder.
Think about where steroid hormones act within the cell compared to peptide hormones. Consider the chemical nature of steroids: are they water-soluble or lipid-soluble? How would this affect their pathway of signaling , especially regarding the cell membrane and gene expression?
3 / 109
Category:
Endo – Physio
What is the action of steroid hormone when it binds to its target cell receptor?
Steroid hormones (like cortisol, aldosterone, estrogen, progesterone, testosterone) are lipophilic (fat-soluble) . Because of this property, they can easily diffuse through the lipid bilayer of the cell membrane .
Mechanism of Action:
Steroid hormone enters the cell by simple diffusion.
It binds to a specific intracellular receptor , either in the cytoplasm or nucleus .
The hormone-receptor complex translocates to the nucleus (if not already there).
It binds to specific DNA sequences called Hormone Response Elements (HREs) .
This activates or represses transcription of target genes, leading to mRNA production and subsequent protein synthesis .
Key Point:
Steroid hormones have genomic effects , meaning they turn specific genes on or off , resulting in slow but long-lasting actions .
Incorrect Answer Explanations:
It causes the formation of releasing hormones
Releasing hormones (like CRH, TRH) are secreted by the hypothalamus to control the anterior pituitary.
Steroid hormones do not induce the formation of releasing hormones .
It causes the formation of cyclic AMP (cAMP)
cAMP is a second messenger primarily used by peptide and protein hormones (like glucagon, FSH, LH, ACTH).
Steroid hormones do not use cAMP ; they act directly on nuclear DNA .
It causes the formation of cyclic GMP (cGMP)
cGMP is involved in specific signal transduction pathways (e.g., nitric oxide signaling, atrial natriuretic peptide).
It is not involved in steroid hormone action .
It is converted into cholesterol, which acts as a second messenger
This is incorrect and biologically impossible .
Cholesterol is the precursor to steroid hormones , not the product.
Steroid hormones are not converted back into cholesterol , and cholesterol is not a second messenger .
Consider the direct physical effects of fat accumulation in the bloodstream . When certain lipid levels become extremely high, they don’t just affect vessels—they can trigger acute organ damage through enzymatic or metabolic complications. Think about which organ deals with digestion and how excess fat might overwhelm its normal function.
4 / 109
Category:
Endo – Pathology
Hypertriglyceridemia is associated with which of the following condition?
When Triglycerides Are Dangerously High (> 1000 mg/dL):
Pancreatitis becomes a significant risk.
The excess triglycerides are broken down by pancreatic lipase into free fatty acids (FFAs) in the pancreatic capillaries.
FFAs are toxic at high concentrations , leading to pancreatic inflammation, ischemia, and cellular injury .
Clinical Relevance:
Hypertriglyceridemia is one of the top three causes of acute pancreatitis , along with gallstones and alcohol abuse .
It is particularly common in patients with uncontrolled diabetes, obesity, and familial lipid disorders .
Incorrect Answer Explanations:
Diabetes Type 1
Type 1 Diabetes Mellitus involves autoimmune destruction of pancreatic beta cells , leading to insulin deficiency .
While diabetic ketoacidosis (DKA) may transiently elevate triglycerides, severe hypertriglyceridemia is more typical of Type 2 diabetes or metabolic syndrome , not Type 1.
Not directly linked to pancreatitis unless DKA-induced hypertriglyceridemia occurs, which is rare.
Retinopathy
Diabetic retinopathy is a microvascular complication of diabetes , primarily related to chronic hyperglycemia , not hypertriglyceridemia.
Lipid abnormalities may worsen retinal outcomes but retinopathy is not directly caused by hypertriglyceridemia .
Diabetic Foot
Diabetic foot ulcers are caused by neuropathy, ischemia, and infection .
There is no direct association with hypertriglyceridemia .
Appendicitis
Appendicitis is due to inflammation of the appendix , commonly from luminal obstruction (like fecaliths or lymphoid hyperplasia).
There is no link between triglyceride levels and appendicitis .
When assessing a patient with slow metabolism-related symptoms , think about how these changes can subtly affect the neuromuscular system , not just the skin, bowels, or weight. Consider how the autonomic nervous system and reflex arcs are influenced by changes in metabolic rate.
5 / 109
Category:
Endo – Physio
A young woman came to the clinic with the complaints of dry skin, constipation and other hypothyroid findings. Which clinical signs will further aid in the diagnosis?
Hypothyroidism slows down many of the body’s processes, including metabolism, neuromuscular activity, and cardiovascular function.
Common Signs and Symptoms of Hypothyroidism:
Weight gain (due to decreased basal metabolic rate)
Dry skin
Constipation (slowed gastrointestinal motility)
Cold intolerance
Fatigue
Bradycardia (slow heart rate)
Depression or cognitive slowing
Menstrual irregularities
Delayed relaxation phase of deep tendon reflexes (especially the ankle jerk)—this is a classic clinical finding and considered one of the most specific neuromuscular signs of hypothyroidism.
Why does this happen?
Delayed relaxation of reflexes is thought to result from impaired calcium reuptake into the sarcoplasmic reticulum of muscle due to thyroid hormone deficiency.
Weight gain results from decreased energy expenditure.
These signs complement the other findings in hypothyroidism and help solidify the diagnosis.
Incorrect Answer Explanations:
Systolic Hypertension and Hyper-reflexia
These are typically seen in hyperthyroidism , not hypothyroidism.
Hyperthyroidism increases sympathetic activity , leading to brisk reflexes and higher systolic pressure due to increased cardiac output.
Palmar Erythema and Tachycardia
These are signs of hyperthyroidism or liver disease , not hypothyroidism.
In hyperthyroidism, increased circulation causes palmar erythema and tachycardia due to heightened metabolic demands.
Lid Lag and Lid Retraction
Weight Loss and Bruit
When choosing a treatment for a patient with both metabolic and reproductive symptoms , think about a medication that can target the root cause instead of just treating the lab numbers. Consider the pathophysiology of insulin resistance and its broader effects on the endocrine system
6 / 109
Category:
Endo – Pharmacology
A female patient came to the clinic with the complaints of amenorrhea, increased blood sugar and obesity. She is not on any medications. Which of the following drug should be advised to her?
Biguanides (Metformin) are the first-line drug for patients with insulin resistance , which is commonly seen in both Type 2 Diabetes Mellitus and Polycystic Ovary Syndrome (PCOS) .
Mechanism of Action:
Reduces hepatic glucose production (gluconeogenesis)
Increases insulin sensitivity in peripheral tissues (muscle and fat)
Helps with weight control or mild weight loss , making it ideal for obese patients
Why is Metformin Ideal Here?
Hyperglycemia: Metformin lowers blood sugar without causing hypoglycemia.
Obesity: Metformin does not cause weight gain , unlike insulin or sulfonylureas.
Amenorrhea: In cases like PCOS, Metformin can help restore menstrual cycles by improving insulin sensitivity, which reduces hyperandrogenism and helps normalize ovulation.
Thus, Metformin addresses all three of the patient’s issues in a safe and effective manner.
Incorrect Answer Explanations:
Alpha-glucosidase Inhibitors
Insulin
Sulphonylureas
Thiazolidinediones (TZDs)
Think about which arthritis medications are known to cause systemic side effects like weight gain, muscle weakness, and visual changes . Could the drug mimic a hormonal syndrome if used for a long time or at high doses?
7 / 109
Category:
Endo – Pharmacology
A 35-year-old female came to the clinic with the complaints of muscle weakness, excessive weight gain and blurry vision. She has been taking medications to relieve her arthritic pain. Which of the following medication is responsible in this case?
Steroids (glucocorticoids like prednisone) are commonly prescribed for rheumatoid arthritis and other inflammatory conditions.
Long-term steroid use can lead to iatrogenic Cushing’s syndrome , which causes:
Weight gain (central obesity, “moon face”, “buffalo hump”)
Muscle weakness (proximal myopathy due to protein catabolism)
Blurry vision (due to steroid-induced cataracts or glaucoma )
Hyperglycemia , osteoporosis , skin thinning , and immune suppression are other known side effects.
Why the Other Options Are Incorrect:
Option
Why It’s Incorrect
Leflunomide
❌ Can cause liver toxicity and teratogenicity , but not Cushingoid symptoms .
Methotrexate
❌ Causes liver toxicity , bone marrow suppression , and mouth ulcers , but not weight gain or vision problems .
Azathioprine
❌ Can lead to bone marrow suppression and infections , but not Cushing-like symptoms .
Sulfasalazine
❌ May cause GI upset, rash, or hemolysis , but not the features described .
Summary:
The patient’s muscle weakness, weight gain, and blurry vision are classic side effects of long-term steroid use , which is a common medication in arthritis management
Consider whether the condition affects glucose metabolism or insulin regulation , or if it’s primarily a structural disorder of an organ unrelated to metabolic control. Does this condition involve cysts in the kidneys , or does it involve vascular, cardiac, or metabolic dysfunction ?
8 / 109
Category:
Endo – Pathology
Which of the following is not associated with diabetes mellitus?
Polycystic kidney disease (PKD) is a genetic disorder that leads to the formation of multiple fluid-filled cysts in the kidneys .
It primarily affects kidney structure and function , leading to:
**PKD is not directly associated with diabetes mellitus or its pathogenesis.
It does not affect insulin production, insulin resistance, or glucose metabolism .
Conditions Associated with Diabetes Mellitus:
Condition
Association with Diabetes
Obesity
✅ Major risk factor for type 2 diabetes due to insulin resistance .
Genetic predisposition
✅ Family history is a key risk factor , especially for type 2 diabetes .
Heart disease
✅ Cardiovascular disease is a major complication of diabetes .
Atherosclerosis
✅ Diabetes accelerates atherosclerosis , increasing the risk of stroke, MI, and PAD .
Why the Other Options Are Incorrect:
Option
Why It’s Incorrect
Obesity
❌ Strongly linked to insulin resistance and type 2 diabetes .
Genetic predisposition
❌ Family history is a well-established risk factor .
Heart disease
❌ Diabetes is a major cause of cardiovascular disease .
Atherosclerosis
❌ Diabetes accelerates atherosclerosis , leading to vascular complications.
Summary:
Polycystic kidney disease is not associated with diabetes mellitus , whereas obesity, genetics, heart disease, and atherosclerosis are closely linked to diabetes and its complications.
Consider the type of thyroiditis that occurs after a viral infection like mumps. This condition typically causes thyroid pain, inflammation, and transient thyroid dysfunction . Does the primary problem involve autoimmunity, infection, or inflammation , and what would be the most logical way to reduce the inflammatory process ?
9 / 109
Category:
Endo – Pathology
A woman developed severe inflammation of the thyroid gland after being infected by mumps. What is the treatment of choice?
Subacute thyroiditis is also called De Quervain’s thyroiditis or granulomatous thyroiditis .
Often follows viral infections such as mumps, influenza, or coxsackievirus .
Characterized by:
Painful, tender thyroid gland
Transient hyperthyroidism (due to release of preformed thyroid hormones)
Possible hypothyroid phase before recovery
Treatment of Choice:
The main goal is to reduce pain and inflammation .
First-line therapy :
Non-steroidal anti-inflammatory drugs (NSAIDs) for mild cases
Glucocorticoids (e.g., prednisone) if NSAIDs are not sufficient or if symptoms are severe
Beta blockers can be used for symptomatic control of hyperthyroidism (e.g., palpitations) but do not treat the inflammation.
Levothyroxine is not required initially , unless the patient becomes hypothyroid long-term .
Anti-thyroid drugs (e.g., methimazole) are not indicated , because the hyperthyroidism is due to hormone leakage , not overproduction.
Why the Other Options Are Incorrect:
Option
Why It’s Incorrect
Levothyroxine
❌ Not needed unless there’s long-term hypothyroidism after the inflammation subsides.
Beta blockers
❌ Help control symptoms of hyperthyroidism but do not treat the underlying inflammation .
Anti-thyroid drugs
❌ Ineffective because thyroid hormone is being released from damaged cells , not overproduced.
NSAIDs alone
❌ May be tried first for mild cases, but in severe cases like this (post-mumps thyroiditis) , steroids are often needed .
Summary:
The treatment of subacute (De Quervain’s) thyroiditis is NSAIDs and steroids , with steroids reserved for severe or refractory cases .
Consider which hormone deficiency would lead to hypotension, electrolyte imbalances, hypoglycemia, and shock-like symptoms . What hormone must be urgently replaced to stabilize the patient , especially if the problem is related to the adrenal glands ?
10 / 109
Category:
Endo – Pharmacology
A 16-year-old girl came to the clinic with the complaints of lethargy and weight loss since six months. She also has hypotension and altered mentation. She also has mild flank tendernes. What is the treatment of choice?
Why IV Hydrocortisone is the Treatment of Choice:
The clinical picture suggests acute adrenal insufficiency (Addisonian crisis) superimposed on chronic adrenal insufficiency .
Hydrocortisone is the first-line treatment because it provides glucocorticoid replacement , which is essential for:
Maintaining blood pressure (by sensitizing vasculature to catecholamines)
Maintaining blood sugar (prevents hypoglycemia)
Reducing inflammation and stress response
In high doses , hydrocortisone also has mineralocorticoid activity , helping with sodium retention and potassium excretion .
Standard Emergency Treatment:
Immediate IV hydrocortisone
IV fluids (normal saline or dextrose) to correct hypotension and dehydration .
Glucose correction if needed (due to hypoglycemia).
Identify and treat any underlying cause (e.g., infection, stopping steroids abruptly).
Why the Other Options Are Incorrect:
Option
Why It’s Incorrect
Insulin
❌ The patient is hypoglycemic , not hyperglycemic, so insulin would worsen her condition.
Antibiotics
❌ Use only if there’s a confirmed infection . The primary issue is adrenal failure , so steroids are priority.
Sedatives
❌ May worsen altered mentation and hypotension . Sedation is dangerous in adrenal crisis.
Fluids alone
❌ Fluids help but are not sufficient by themselves . Without steroid replacement , shock will persist.
Summary:
In a 16-year-old girl with Addisonian crisis (hypotension, lethargy, hypoglycemia, and flank tenderness) , the treatment of choice is intravenous hydrocortisone to correct glucocorticoid deficiency and stabilize the patient .
Think about how you would directly assess adrenal function . Is it enough to measure a single hormone level at one time , or should you stimulate the adrenal glands to see if they can respond appropriately?
11 / 109
Category:
Endo – Physio
What is the most appropriate investigation in a patient with suspected adrenal insufficiency?
Why the Short Synacthen Test is Best:
The Short Synacthen Test (ACTH stimulation test) is the gold standard initial diagnostic test for suspected adrenal insufficiency .
Synacthen is a synthetic ACTH .
Procedure:
Measure baseline cortisol .
Administer Synacthen (250 µg IV or IM) .
Measure cortisol again at 30 and 60 minutes .
Normal response : Cortisol rises significantly (>18–20 µg/dL).
Adrenal insufficiency : Little or no cortisol rise after Synacthen.
Why It’s Important:
Why the Other Options Are Incorrect:
Option
Why It’s Incorrect
Potassium levels
❌ May show hyperkalemia in primary adrenal insufficiency , but this is not diagnostic .
Adrenal antibodies
❌ Helpful if autoimmune Addison’s disease is suspected, but only after functional testing is done .
Serum adrenal androgens
❌ DHEA and other adrenal androgens are not the primary focus in adrenal insufficiency diagnosis.
Random plasma cortisol
❌ Cortisol levels vary throughout the day and with stress. A random cortisol is not reliable alone for diagnosis.
Summary:
The Short Synacthen Test is the most appropriate initial investigation in a patient with suspected adrenal insufficiency , as it directly assesses adrenal cortisol production capacity .
Consider the adrenal glands’ role in maintaining blood pressure, electrolyte balance, and energy metabolism . When these glands fail, patients may present with hypotension, weight loss, and fatigue . Could the flank tenderness be related to the adrenal glands , which sit just above the kidneys?
12 / 109
Category:
Endo – Pathology
A 16-year-old girl came to the clinic with the complaints of lethargy and weight loss since six months. She also has hypotension and altered mentation. She also has mild flank tenderness. What is the probable diagnosis?
What is Addison’s Disease?
Addison’s disease is primary adrenal insufficiency —the adrenal glands fail to produce cortisol and aldosterone .
Common causes include autoimmune destruction , infections (e.g., TB), or adrenal hemorrhage .
Key Clinical Features in This Case:
Symptom
Explanation
Lethargy, fatigue
Due to low cortisol levels
Weight loss
Due to catabolic state and anorexia
Hypotension
Due to aldosterone deficiency → salt loss → volume depletion
Altered mentation
Due to hypoglycemia or electrolyte imbalances (e.g., hyponatremia, hyperkalemia)
Flank tenderness
May be from enlarged or inflamed adrenal glands (e.g., TB, hemorrhage), though usually mild
Lab Findings Typically Seen:
Why the Other Options Are Incorrect:
Option
Why It’s Incorrect
Pyelonephritis
❌ Typically causes fever, urinary symptoms, and severe flank pain , not chronic weight loss, hypotension, or altered mentation .
Phaeochromocytoma
❌ Causes hypertension , not hypotension . Symptoms include headache, palpitations, and sweating , not weight loss with low BP.
Hypothyroidism
❌ Causes weight gain , bradycardia , and cold intolerance , not weight loss and hypotension .
Diabetic ketoacidosis (DKA)
❌ DKA presents acutely with polyuria, vomiting, abdominal pain, Kussmaul breathing , and high blood sugar —not chronic lethargy and weight loss.
Summary:
A 16-year-old girl with chronic weight loss, fatigue, hypotension, altered mental status, and flank tenderness most likely has Addison’s disease (primary adrenal insufficiency) .
Think about syndromes that involve tumors in multiple endocrine glands . One of them is known as the “3 Ps ” syndrome because it typically affects the Parathyroid, Pancreas, and Pituitary glands . Could this explain a combination of hypoglycemia, kidney stones, and hormonal dysfunction ?
13 / 109
Category:
Endo – Pathology
A young male came to the clinic with hypoglycemia and loose stools. He also has renal stones and erectile dysfunction. He does not have diabetes. What is the most likely diagnosis?
What is MEN1?
Multiple Endocrine Neoplasia Type 1 (MEN1) is a hereditary syndrome characterized by tumors in:
Organ
Common Tumor
Parathyroid glands
Parathyroid hyperplasia/adenoma → Hypercalcemia → Renal stones
Pancreas (or GI tract)
Gastrinomas, Insulinomas, VIPomas → Hypoglycemia, diarrhea/loose stools
Pituitary gland
Pituitary adenomas → May cause prolactinoma → erectile dysfunction
Symptoms in This Case:
Hypoglycemia → Suggests insulinoma (pancreatic islet cell tumor)
Loose stools → May be due to VIPoma or gastrinoma (also seen in MEN1)
Renal stones → Due to hyperparathyroidism causing hypercalcemia
Erectile dysfunction → Could be from pituitary tumor (prolactinoma) affecting gonadal function
Why the Other Options Are Incorrect:
Option
Why It’s Incorrect
MEN2 (MEN2A/MEN2B)
❌ Involves medullary thyroid carcinoma, pheochromocytoma, and parathyroid hyperplasia (MEN2A). Hypoglycemia and diarrhea are not typical .
MEN3 (MEN2B)
❌ Characterized by mucosal neuromas, marfanoid habitus, pheochromocytoma, and medullary thyroid cancer . No hypoglycemia or renal stones.
Schmidt’s syndrome
❌ This is autoimmune polyglandular syndrome type 2 , involving Addison’s disease, thyroid disease, and type 1 diabetes , not endocrine tumors.
Autoimmune polyendocrinopathy type 1
❌ Involves Addison’s disease, hypoparathyroidism, and mucocutaneous candidiasis , not tumors or hyperparathyroidism.
Summary:
The combination of hypoglycemia, diarrhea, renal stones, and erectile dysfunction is characteristic of MEN1 , which involves tumors of the parathyroid, pancreas, and pituitary .
Consider which small glands in the neck are often accidentally damaged or removed during thyroid or neck surgeries . These glands regulate calcium homeostasis , and their loss can lead to low calcium , causing neurological symptoms like tingling or numbness .
14 / 109
Category:
Endo – Physio
A male came to the outpatient department with the complaint of tingling sensation in his body some time after a neck surgery. Which of the following is the most likely reason for his complaint?
Why Parathyroid Gland Removal Is the Most Likely Cause:
The parathyroid glands regulate calcium levels through the secretion of parathyroid hormone (PTH) .
Neck surgeries , especially thyroidectomy , may accidentally damage or remove the parathyroid glands because of their close anatomical location .
Hypocalcemia due to parathyroid removal leads to:
Tingling (paresthesia) around the mouth, fingers, and toes
Muscle cramps
Tetany (severe hypocalcemia)
Chvostek’s and Trousseau’s signs may be positive on physical exam
Why This Happens:
Low PTH → Decreased calcium reabsorption from bones, kidneys, and intestines
Result: Hypocalcemia → Neuromuscular excitability → Tingling and cramps
Why the Other Options Are Incorrect:
Option
Why It’s Incorrect
Diet restriction
❌ May cause long-term deficiencies , but does not cause acute tingling right after neck surgery .
Sympathetic trunk damage
❌ Can cause Horner’s syndrome (ptosis, miosis, anhidrosis), not tingling or paresthesia .
Thyroid surgery
❌ Thyroid surgery alone is not the direct cause; the symptom results from parathyroid damage/removal during thyroid surgery .
Psychological
❌ Anxiety may cause subjective tingling , but after neck surgery, hypocalcemia must be ruled out first , especially with objective signs.
Summary:
The most likely cause of tingling after neck surgery is accidental removal or damage to the parathyroid glands , leading to hypocalcemia and neuromuscular symptoms .
When evaluating thyroid function, think about which hormone provides the most sensitive indication of thyroid dysfunction and which hormone reflects the actual thyroid hormone output affecting the body’s metabolism . Why is it important to check both?
15 / 109
Consider which anti-thyroid drug can lead to a serious side effect involving the bone marrow , resulting in low white blood cell count . This condition can present suddenly with sore throat and fever , making it a medical emergency .
16 / 109
Category:
Endo – Pharmacology
Which of the following anti-thyroid drug is responsible for causing sore throat with fever? Total leukocyte count is also low.
Methimazole is a thionamide anti-thyroid drug used in hyperthyroidism treatment.
Its mechanism : Inhibits thyroid peroxidase , preventing the organification of iodine and thyroid hormone synthesis .
Serious Side Effect:
Agranulocytosis is the most feared side effect of methimazole (can also occur with propylthiouracil but is more common with methimazole).
It involves severe reduction in neutrophils (granulocytes) → very low WBC count .
Presents as:
Management:
Why the Other Options Are Incorrect:
Option
Why It’s Incorrect
Propylthiouracil
❌ Can also cause agranulocytosis, but methimazole is the more common culprit . Propylthiouracil is also associated with liver toxicity .
Levothyroxine
❌ Thyroid hormone replacement , not an anti-thyroid drug. It does not cause agranulocytosis.
Radioiodine
❌ Can lead to hypothyroidism or thyroiditis , but does not cause agranulocytosis .
Liothyronine
❌ A synthetic T3 preparation used for hypothyroidism or myxedema coma—not related to WBC suppression.
Summary:
Methimazole can cause agranulocytosis , presenting as sore throat, fever, and a low WBC count —a medical emergency requiring immediate attention.
Consider a condition where reproductive hormone deficiency is combined with loss of the sense of smell . Could there be a developmental link between the olfactory system and the hypothalamic GnRH neurons ?
17 / 109
Category:
Endo – Embryology
A 10-year-old presented to the outpatient department with the complaint of anosmia. He has undescended testes and shows signs of hypogonadism. What is the most likely diagnosis?
What is Kallmann’s Syndrome?
Kallmann’s syndrome is a form of hypogonadotropic hypogonadism combined with anosmia (loss of smell) .
It results from defective migration of GnRH-secreting neurons and olfactory neurons during embryonic development.
Pathophysiology:
Normally, GnRH neurons originate near the olfactory placode and migrate to the hypothalamus .
In Kallmann’s syndrome, this migration fails → deficient GnRH production .
This leads to:
Low LH/FSH (hypogonadotropic hypogonadism)
Undescended testes (cryptorchidism) or underdeveloped genitalia in males
Delayed or absent puberty
Anosmia or hyposmia (impaired smell)
Why the Other Options Are Incorrect:
Option
Why It’s Incorrect
McCune-Albright syndrome
❌ Causes precocious puberty , café-au-lait spots, and fibrous dysplasia—not anosmia or hypogonadism .
Delayed puberty
❌ Delayed puberty can be constitutional but is not associated with anosmia or undescended testes .
Turner Syndrome
❌ Occurs in females (45,XO), not in a 10-year-old male . Features include short stature, webbed neck, and streak ovaries—not anosmia.
Androgen Insensitivity Syndrome
❌ Patients are genetically male (46,XY) but have female external genitalia and normal or high androgen levels . Anosmia is not a feature .
Summary:
A 10-year-old boy with anosmia, undescended testes, and hypogonadism most likely has Kallmann’s syndrome , a developmental disorder of GnRH and olfactory neuron migration .
Consider how the body naturally regulates blood glucose after meals . There are incretin hormones (like GLP-1) that help stimulate insulin release and suppress glucagon , but they are rapidly broken down by a specific enzyme . Sitagliptin works by targeting this enzyme . What is that enzyme?
18 / 109
Category:
Endo – Pharmacology
What is the mechanism of action of sitagliptin?
Mechanism of Action:
DPP-4 normally breaks down incretin hormones (GLP-1 and GIP).
Incretins increase insulin secretion (glucose-dependent) and decrease glucagon secretion after meals.
Sitagliptin inhibits DPP-4 , leading to:
Increased GLP-1 and GIP levels
Prolonged insulin secretion
Decreased glucagon release
Better postprandial glucose control
Why the Other Options Are Incorrect:
Option
Why It’s Incorrect
PPARγ agonists
❌ These are thiazolidinediones (e.g., pioglitazone), which increase insulin sensitivity , not inhibit DPP-4.
SGLT2 inhibitor
❌ These drugs (e.g., empagliflozin ) block renal glucose reabsorption , causing glucose loss in urine , not affecting incretins.
Alpha-glucosidase inhibitor
❌ These drugs (e.g., acarbose ) delay carbohydrate absorption in the intestine , unrelated to incretin pathways.
GLP-1 receptor agonist
❌ Drugs like exenatide and liraglutide mimic GLP-1 , but sitagliptin prolongs natural GLP-1 action by inhibiting DPP-4 .
Summary:
Sitagliptin is a DPP-4 inhibitor that increases incretin levels , leading to improved insulin secretion and reduced glucagon , helping control blood sugar in type 2 diabetes.
Consider the effects of severe postpartum hemorrhage on the pituitary gland , which is highly vascular and vulnerable to ischemia after blood loss . Why would a woman stop menstruating and be unable to breastfeed after delivery?
19 / 109
Category:
Endo – Pathology
A 28-year-old woman who gave birth recently came to the outpatient department with the complaints of amenorrhea and inability to breastfeed her child. She gives a past history of postpartum haemorrhage. What is your diagnosis based on her complaints?
Sheehan’s syndrome is postpartum hypopituitarism caused by ischemic necrosis of the anterior pituitary due to severe blood loss during or after childbirth (postpartum hemorrhage).
The anterior pituitary enlarges during pregnancy , making it more susceptible to ischemia in the event of hypotension or hemorrhage .
Clinical Features:
Failure to lactate (due to prolactin deficiency )
Amenorrhea or oligomenorrhea (due to gonadotropin deficiency → low LH/FSH )
Fatigue, weakness, cold intolerance (due to TSH and ACTH deficiency )
Loss of pubic and axillary hair (optional finding)
Why the Other Options Are Incorrect:
Option
Why It’s Incorrect
Adrenal insufficiency
May cause fatigue, but would not explain inability to breastfeed or amenorrhea after postpartum hemorrhage directly.
Prolactinoma
Causes galactorrhea , not inability to lactate. Prolactin would be high , not low.
Hypothyroidism
Can cause amenorrhea , but not typically failure of lactation immediately postpartum due to pituitary damage.
Hyperparathyroidism
Involves calcium regulation , unrelated to lactation or menstrual cycles.
Summary:
A postpartum woman with amenorrhea, inability to lactate, and a history of postpartum hemorrhage is classic for Sheehan’s syndrome (postpartum pituitary necrosis) .
Think about what hormone is responsible for milk production (lactation) . If this hormone remains elevated when a woman is not breastfeeding , it can disrupt normal reproductive function by interfering with the secretion of GnRH , leading to infertility . What hormone could cause both galactorrhea and infertility ?
20 / 109
Category:
Endo – Physio
A 31-year-old woman came to the gynaecology outpatient department with the complaint of milk production from her breasts. She has been married for six years and is unable to conceive. Which lab investigation must the doctor advise to diagnose her condition?
Why Prolactin is the Most Important Test:
Prolactin is the hormone responsible for milk production (galactorrhea) .
Hyperprolactinemia (excess prolactin) can cause:
Galactorrhea (milk secretion without pregnancy or breastfeeding)
Infertility due to suppression of GnRH , leading to low LH/FSH and anovulation
The most common cause is a prolactinoma (pituitary adenoma) , but other causes include:
Why the Other Options Are Incorrect:
Option
Why It’s Incorrect
Cortisol
Related to stress and metabolism , not milk production or reproductive suppression.
LH/FSH
May be low due to high prolactin , but checking prolactin directly is more specific for this case.
TSH
Hypothyroidism can increase prolactin (via TRH stimulation), but prolactin must still be measured directly to confirm the diagnosis .
Serum estrogen
Estrogen levels may be affected by anovulation , but prolactin is the primary test for galactorrhea with infertility .
Summary:
In a woman with galactorrhea and infertility , the first lab test to order is serum prolactin to evaluate for hyperprolactinemia , which is the most likely cause.
Consider which common endocrine disorder in young women presents with menstrual irregularities, hirsutism, and weight gain . It is also frequently associated with insulin resistance and metabolic syndrome . What condition fits this triad?
21 / 109
Category:
Endo – Pathology
A 21-year-old unmarried girl presented to the gynecology outpatient department with complaints of oligomenorrhea, excessive hair growth, and significant weight gain for the past year. What is the most probable diagnosis based on the presenting complaints?
Why PCOS is the Most Probable Diagnosis:
Polycystic Ovarian Syndrome (PCOS) is the most common endocrine disorder in reproductive-aged women , typically presenting with:
Oligomenorrhea or amenorrhea (due to anovulation)
Hirsutism (excess hair growth) from androgen excess
Weight gain/obesity , often linked to insulin resistance
Polycystic ovaries seen on ultrasound (multiple peripheral follicles)
Pathophysiology:
Increased LH:FSH ratio , leading to increased androgen production by the ovaries
Impaired follicle maturation → anovulation
Insulin resistance worsens hormonal imbalance
Why the Other Options Are Incorrect:
Option
Why It’s Incorrect
Acromegaly
Presents with coarse facial features, enlarged hands/feet, and jaw growth , not primarily with menstrual issues and hirsutism .
Hypothyroidism
Can cause weight gain and menstrual problems , but hirsutism is not a typical feature .
Hyperprolactinemia
Usually presents with amenorrhea and galactorrhea , not hirsutism or weight gain .
Congenital adrenal hyperplasia (CAH)
CAH can cause hirsutism and menstrual irregularities , but usually presents earlier in life and often with ambiguous genitalia or salt-wasting in severe forms .
Summary:
The combination of oligomenorrhea, hirsutism, and weight gain in a young woman is most consistent with Polycystic Ovarian Syndrome (PCOS).
Consider the feedback loop between PTH and calcium . Normally, high calcium levels suppress PTH secretion . If PTH remains elevated despite high calcium , where is the problem likely located—in the parathyroid glands themselves , or in a compensatory response to another condition?
22 / 109
Category:
Endo – Pathology
High PTH, high plasma calcium, and normal levels of creatinine and vitamin D are consistent with which of the following parathyroid states?
Understanding the Lab Findings:
Parameter
Finding
PTH (Parathyroid Hormone)
High
Plasma Calcium
High
Creatinine
Normal
Vitamin D
Normal
Primary Hyperparathyroidism:
Caused by parathyroid adenoma (most common) , parathyroid hyperplasia , or rarely parathyroid carcinoma .
In this condition, the parathyroid glands secrete PTH autonomously , ignoring normal feedback mechanisms .
High PTH leads to:
Increased calcium reabsorption from bone and kidney
Increased activation of Vitamin D (but normal Vitamin D here suggests early stage or regulated synthesis)
Increased intestinal absorption of calcium (via Vitamin D)
Result: High calcium, high PTH , and normal renal function (normal creatinine)
Why the Other Options Are Incorrect:
Option
Explanation
Primary hypoparathyroidism
❌ Would cause low PTH and low calcium , not high levels.
Secondary hyperparathyroidism
❌ PTH is high, but calcium is usually low or normal , due to chronic conditions like renal failure or vitamin D deficiency .
Tertiary hyperparathyroidism
❌ Occurs in chronic renal failure , with high PTH and high calcium , but typically with elevated creatinine (renal dysfunction), which is normal here .
Secondary hypoparathyroidism
❌ Would cause low PTH due to external suppression (e.g., surgery, radiation), not high PTH.
Summary:
High PTH + High Calcium + Normal Creatinine and Vitamin D is classic for Primary Hyperparathyroidism .
Think about the geographic factors that influence thyroid health . In certain regions, especially those far from the sea or in mountainous areas , an important nutrient required for thyroid hormone synthesis is often lacking. What is this nutrient?
23 / 109
Category:
Endo – Community Medicine/Behavioral Sciences
A 25-year-old man presents with features of hypothyroidism. He lives in a mountainous area. Which of these is the most likely cause of his condition?
Iodine is essential for the production of thyroid hormones (T3 and T4) .
People living in mountainous or inland areas often have limited access to iodine-rich foods , such as seafood.
Soil and water in these areas are typically iodine-deficient , leading to iodine deficiency disorders (IDD) .
Hypothyroidism from Iodine Deficiency:
Lack of iodine → Reduced thyroid hormone production
Leads to hypothyroidism and sometimes goiter (thyroid enlargement as compensation)
Common in regions without iodized salt programs
Why the Other Options Are Incorrect:
Option
Why It’s Incorrect
Drug use
Some drugs (e.g., lithium, amiodarone) can cause hypothyroidism, but the question emphasizes geography, not medication .
Thyroid tumor
Tumors usually cause nodules or hyperthyroidism (if functioning), or local compressive symptoms, but are not the most common cause in this context .
Autoimmunity
Hashimoto’s thyroiditis is a common cause in developed regions, but iodine deficiency is more likely in mountainous or endemic areas .
Iodine excess
Excess iodine can lead to hypothyroidism (Wolff-Chaikoff effect) , but that is rare , and in a mountainous area, deficiency is more probable .
Summary:
In a mountainous region, iodine deficiency is the most common cause of hypothyroidism due to low environmental iodine availability.
Consider what hormone is secreted when you’re dehydrated or have a high blood osmolality . This hormone acts on the kidneys , not to retain salts, but to retain water and concentrate urine . What is its main goal in such situations?
24 / 109
Think about where glycogen breakdown begins . Does the process start by removing glucose units from the branches themselves , or does it first work on the linear chains from the non-reducing ends ?
25 / 109
Category:
Endo – Biochemistry
Which of the following enzymes is responsible for cleaving off glucose-1-phosphate from the peripheral end of a long branch of glycogen during glycogenolysis?
Glycogenolysis is the process of breaking down glycogen into glucose-1-phosphate for energy production.
It starts at the non-reducing ends of glycogen , removing glucose residues one at a time .
Role of Glycogen Phosphorylase:
Glycogen phosphorylase cleaves α-1,4 glycosidic bonds at the peripheral (non-reducing) ends of glycogen.
It uses inorganic phosphate (Pi) to release glucose-1-phosphate .
This is the rate-limiting step of glycogenolysis .
What Happens to Branches?
When glycogen phosphorylase gets close to a branch point, it cannot cleave α-1,6 linkages .
At that point, the debranching enzyme takes over to handle branch points.
Why the Other Options Are Incorrect:
Option
Why It’s Incorrect
Glucose-6-phosphatase
Removes the phosphate from glucose-6-phosphate to produce free glucose , mostly in the liver , but it does not cleave glycogen .
Branching enzyme
Adds α-1,6 branches during glycogenesis , not involved in glycogen breakdown.
Debranching enzyme
Removes α-1,6 linkages at branch points but does not remove glucose-1-phosphate from linear chains .
Glycogen synthase
Builds glycogen by forming α-1,4 linkages , the opposite of glycogenolysis .
Summary:
Glycogen phosphorylase is responsible for cleaving glucose-1-phosphate from the peripheral ends of glycogen during glycogenolysis .
Think about what happens when the body is under severe stress to produce more red blood cells , especially in hemolytic diseases . Does the bone marrow have time to let all the cells mature fully before releasing them, or does it push out immature forms ?
26 / 109
Category:
Endo – Pathology
What is released from the bone marrow in excess amounts in erythroblastosis fetalis?
Erythroblastosis fetalis (also known as hemolytic disease of the newborn ) occurs when the maternal immune system destroys fetal red blood cells , often due to Rh incompatibility .
The fetal body responds by overproducing red blood cell precursors to replace the destroyed cells.
Bone Marrow Response:
Normally, the bone marrow releases mature red blood cells (RBCs) .
In erythroblastosis fetalis , there is massive hemolysis , so the bone marrow becomes hyperactive .
It starts to release immature red blood cell precursors (blast cells) , specifically erythroblasts , into the bloodstream.
This is why the condition is called erythroblastosis (“erythro-” = red blood cells, “-blastosis” = immature cell proliferation).
Why the Other Options Are Incorrect:
Option
Explanation
Red blood cells
Mature RBCs are normally released , but in erythroblastosis fetalis, immature forms (blasts) are released in excess .
Megakaryocytes
These are platelet precursors , unrelated to erythrocyte production.
Lymphocytes
White blood cells , not involved in the compensatory response to hemolysis.
Macrophages
Part of the immune system , not directly produced in response to anemia or hemolysis.
Summary:
In erythroblastosis fetalis, the bone marrow releases blast cells (erythroblasts) into the circulation in excess to compensate for severe red cell destruction.
Consider the solubility of steroid hormones . Are they water-soluble like peptide hormones or lipid-soluble , allowing them to pass through the cell membrane ? Where would a receptor need to be located to detect a hormone that can freely enter the cell?
27 / 109
Category:
Endo – Physio
Steroid hormones bind to which type of receptors?
Steroid hormones are lipophilic (fat-soluble) .
They easily diffuse through the cell membrane .
Once inside the cell, they bind to intracellular receptors , primarily located in the cytoplasm (some may be nuclear).
Mechanism of Action:
Steroid hormone enters the cell .
It binds to a cytoplasmic receptor , forming a hormone-receptor complex .
This complex translocates to the nucleus .
It binds to specific DNA sequences (hormone response elements) to regulate gene transcription .
New proteins are synthesized, leading to long-lasting effects .
Examples of Steroid Hormones:
Cortisol, Aldosterone, Estrogen, Testosterone, Progesterone, Vitamin D
Why the Other Options Are Incorrect:
Option
Explanation
G-protein linked receptor
❌ Used by peptide hormones (e.g., glucagon, epinephrine). Steroid hormones do not use G-protein receptors .
Second messenger
❌ Steroid hormones directly affect gene transcription , not second messenger pathways like cAMP or IP₃/DAG.
Cell surface receptors
❌ Peptide and catecholamine hormones bind cell surface receptors , but steroids cross the membrane .
Enzyme-linked receptor
❌ Used by hormones like insulin (tyrosine kinase receptor) , not by steroids.
Summary:
Steroid hormones bind to cytoplasmic receptors , leading to direct regulation of gene expression .
Consider which part of the autonomic nervous system is responsible for the “fight or flight” response —increasing heart rate, dilating pupils, mobilizing glucose, and redirecting blood to muscles. Which gland’s hormones mimic this system’s effects?
28 / 109
Think about second messenger systems . When certain hormones bind to G-protein coupled receptors (Gs type) , which enzyme is responsible for converting a high-energy molecule into a messenger molecule that activates protein kinase A ?
29 / 109
Category:
Endo – Biochemistry
Which of the following statement is correct regarding adenylate cyclase?
Adenylate cyclase is a membrane-bound enzyme that plays a central role in the cAMP second messenger system .
It is activated by Gs proteins when certain hormones or neurotransmitters bind to their receptors.
It catalyzes the conversion of ATP to cyclic AMP (cAMP) .
Reaction:
ATP→cAMP+PPi (pyrophosphate)ATP→cAMP+PPi (pyrophosphate)
Function of cAMP:
cAMP activates protein kinase A (PKA)
PKA phosphorylates various enzymes, leading to cellular responses such as:
Why the Other Options Are Incorrect:
Option
Explanation
It converts cAMP to ATP
❌ False. The reaction is one-way: ATP → cAMP , not the reverse.
It leads to formation of PIP2
❌ False. PIP2 is involved in the IP₃/DAG pathway , controlled by phospholipase C , not adenylate cyclase.
It converts cAMP to ADP
❌ False. cAMP is not converted to ADP . cAMP is eventually degraded by phosphodiesterase to AMP .
It has the same action as phospholipase
❌ False. Phospholipase C cleaves PIP2 into IP₃ and DAG , a completely different pathway from adenylate cyclase.
Summary:
Adenylate cyclase converts ATP into cAMP , which serves as a second messenger in many hormone signaling pathways.
Consider the difference between how steroid hormones and peptide hormones work . One group usually triggers second messenger systems like cAMP, while the other goes directly into the cell to regulate gene transcription . Which mechanism is NOT typical for steroids ?
30 / 109
Category:
Endo – Physio
Which of the following is not a characteristic of steroids?
Mechanism of Action of Steroid Hormones:
Steroid hormones include:
Glucocorticoids (e.g., cortisol)
Mineralocorticoids (e.g., aldosterone)
Sex hormones (e.g., estrogen, testosterone, progesterone)
Vitamin D (acts like a steroid hormone)
Steroid Hormones Act By:
Crossing the cell membrane (lipid-soluble)
Binding to intracellular receptors (in the cytoplasm or nucleus)
Forming a steroid-hormone receptor complex
This complex binds to DNA regulatory elements (hormone response elements, HREs)
Regulating gene transcription , leading to new protein synthesis.
Why “Activates adenyl cyclase” is Incorrect:
Adenylyl cyclase activation → cAMP pathway is used by peptide and catecholamine hormones , not steroids.
Examples of hormones that use this pathway:
Glucagon, ACTH, TSH, FSH, LH, Epinephrine (via beta receptors)
Steroids do not activate second messenger systems like cAMP directly .
Why the Other Options Are Correct (Characteristics of Steroids):
Option
Explanation
Bind to intracellular receptors
✅ True. This is the primary mode of action of steroids.
Takes part in gene regulation
✅ True. Steroid hormones directly influence gene transcription .
Form steroid-hormone receptor complex
✅ True. The hormone binds to its receptor to form an active complex .
Part of DNA-regulatory elements
✅ True. The hormone-receptor complex binds to DNA at specific response elements .
Summary:
Steroid hormones do not activate adenyl cyclase or use cAMP pathways . They act by binding intracellular receptors and regulating gene transcription , making “Activates adenyl cyclase” the correct answer
Consider the biosynthetic pathway of catecholamines (dopamine, norepinephrine, epinephrine) . Which amino acid serves as the starting material for dopamine , the first catecholamine in the synthesis chain ?
31 / 109
Category:
Endo – Biochemistry
Which of the following is a precursor for catecholamine?
The catecholamines include:
All are synthesized from Tyrosine , which is the direct precursor .
Steps in Catecholamine Synthesis:
Tyrosine → L-DOPA (enzyme: tyrosine hydroxylase; rate-limiting step)
L-DOPA → Dopamine
Dopamine → Norepinephrine
Norepinephrine → Epinephrine (in the adrenal medulla, using phenylethanolamine-N-methyltransferase, stimulated by cortisol)
Why the Other Options Are Incorrect:
Option
Why It’s Incorrect
Tryptophan
Precursor for serotonin and melatonin , not catecholamines.
Glycine
Precursor for porphyrins (heme) and acts as an inhibitory neurotransmitter , not for catecholamines.
Phentolamine
A drug (alpha-adrenergic blocker) , not a precursor.
Alanine
Involved in the glucose-alanine cycle , not catecholamine synthesis.
Summary:
Tyrosine is the precursor of catecholamines , making it the correct answer.
Think about the body’s response to stress . Both cortisol and this hormone are part of the fight-or-flight response , working together to increase blood glucose levels, promote lipolysis, and support cardiovascular function . Which catecholamine shares overlapping metabolic actions with cortisol?
32 / 109
Category:
Endo – Physio
Which of the following hormone has cortisol-like action
Increases blood glucose (gluconeogenesis, decreased peripheral glucose uptake)
Promotes lipolysis (fat breakdown for energy)
Promotes protein catabolism
Supports the vascular system (increases responsiveness to catecholamines)
Suppresses the immune system
How Epinephrine Has Cortisol-like Actions:
Epinephrine , secreted by the adrenal medulla , shares several metabolic actions with cortisol :
Both cortisol and epinephrine are critical for the stress response , though they act via different receptors and pathways (cortisol via intracellular receptors, epinephrine via adrenergic receptors).
Why the Other Options Are Incorrect:
Option
Why It’s Incorrect
Glucagon
Raises blood glucose but does not mimic cortisol’s full range of actions , especially on fat and protein metabolism.
Somatostatin
Inhibits the release of many hormones (like GH, insulin, glucagon) but does not raise glucose or mimic cortisol’s metabolic effects .
Insulin
Opposes cortisol’s effects by promoting glucose uptake and storage .
Norepinephrine
Primarily affects blood pressure (vasoconstriction) with less metabolic action compared to epinephrine.
Summary:
Epinephrine is the hormone that has cortisol-like actions , especially in terms of increasing blood glucose and promoting lipolysis , making it the correct answer.
Consider which anterior pituitary cell type is responsible for producing prolactin . A prolactinoma is a type of pituitary adenoma , not a tumor of the target organ. Which specific pituitary cells secrete prolactin?
33 / 109
Category:
Endo – Pathology
Prolactinoma is a tumor of which of the following?
A prolactinoma is the most common type of pituitary adenoma .
It arises from mammotropes (also called lactotrophs) , the prolactin-secreting cells of the anterior pituitary .
Effects of a Prolactinoma:
Hyperprolactinemia → leads to:
Galactorrhea (inappropriate milk production)
Amenorrhea (in women)
Infertility (in both sexes)
Hypogonadism (due to prolactin’s inhibitory effect on GnRH)
Mass effect symptoms if large (headache, visual field defects from optic chiasm compression)
Why the Other Options Are Incorrect:
Option
Reason
Gonadotropes
Secrete LH and FSH , not prolactin.
Breast
The breast is a target organ of prolactin, but a prolactinoma originates in the pituitary , not in breast tissue.
Hypothalamus
The hypothalamus regulates prolactin via dopamine (inhibitory control) , but prolactin is produced by the anterior pituitary .
Somatotropes
Secrete growth hormone (GH) , not prolactin.
Summary:
A prolactinoma is a tumor of the mammotropes (lactotrophs) in the anterior pituitary , which produce prolactin .
Think about where neurohormones like oxytocin and vasopressin are synthesized versus where they are stored and released . Which brain structure produces these hormones but does not directly release them into circulation ?
34 / 109
Category:
Endo – Physio
Oxytocin and vasopressin are produced by what structure?
Synthesis and Release of Oxytocin and Vasopressin:
Both oxytocin and vasopressin (antidiuretic hormone, ADH) are synthesized in the hypothalamus , specifically in the:
After synthesis, the hormones are transported down the axons of hypothalamic neurons to the posterior pituitary (neurohypophysis) , where they are stored and released into the bloodstream .
Key Concept:
Step
Location
Hormone synthesis
Hypothalamus (paraventricular and supraoptic nuclei)
Hormone storage & release
Posterior pituitary
Why the Other Options Are Incorrect:
Option
Reason
Thalamus
The thalamus is involved in sensory relay , not hormone production.
Posterior pituitary
The posterior pituitary stores and releases oxytocin and vasopressin , but does not produce them .
None of these
Incorrect because the hypothalamus is the correct answer .
Anterior pituitary
The anterior pituitary produces different hormones (e.g., ACTH, TSH, GH) but not oxytocin or vasopressin .
Summary:
Oxytocin and vasopressin are produced in the hypothalamus , then transported to the posterior pituitary for storage and release .
Consider the type of receptor each hormone binds to . Some hormones activate G-protein coupled receptors (GPCRs) linked to adenylyl cyclase and cAMP , while others use different signaling pathways like IP₃/DAG.
35 / 109
Category:
Endo – Physio
Which hormone does not use cAMP as the second messenger?
Hormones Using cAMP as Second Messenger:
The cAMP pathway is one of the most common second messenger systems used by peptide hormones. These hormones bind to G-protein coupled receptors (Gs) , activating adenylyl cyclase , which converts ATP to cAMP , triggering downstream effects.
Examples that use cAMP:
Hormone
Pathway
Glucagon
Activates cAMP in the liver to stimulate glycogenolysis and gluconeogenesis .
FSH
Uses cAMP to mediate its effects on the ovaries and testes .
Calcitonin
Uses cAMP to decrease blood calcium levels by inhibiting bone resorption.
ACTH
Uses cAMP to stimulate cortisol production in the adrenal cortex .
Why Oxytocin Is the Correct Answer (Does Not Use cAMP):
Oxytocin works through the IP₃/DAG pathway , not the cAMP system.
It binds to Gq protein-coupled receptors , activating phospholipase C (PLC) .
This leads to:
The rise in calcium causes smooth muscle contraction in the uterus and mammary glands .
Why the Other Options Are Incorrect (They DO Use cAMP):
Hormone
Why It Uses cAMP
Glucagon
✅ Activates cAMP to raise blood glucose.
FSH
✅ Uses cAMP to stimulate gonadal function.
Calcitonin
✅ Activates cAMP to inhibit osteoclasts and reduce calcium.
ACTH
✅ Stimulates cortisol production via cAMP.
Summary:
Oxytocin does NOT use cAMP ; it works through the IP₃/DAG pathway and calcium-mediated signaling .
Think about which gland stores its hormones extracellularly in follicles , rather than immediately releasing them. Which endocrine organ is unique for having this kind of storage and release mechanism ?
36 / 109
Category:
Endo – Histology
Which gland has follicles with colloid?
The thyroid gland is unique among endocrine glands because it stores its hormone precursors in follicles filled with colloid .
Structure of the Thyroid Gland:
Thyroid follicles are spherical units lined by follicular (epithelial) cells .
The central cavity of each follicle is filled with colloid , a protein-rich material.
Colloid contains thyroglobulin , which is the precursor for thyroid hormones (T3 and T4) .
Why is this unique?
The thyroid is the only endocrine gland that stores large amounts of hormone precursor (thyroglobulin) extracellularly .
Most other endocrine glands store hormones intracellularly in secretory granules .
Why the Other Options Are Incorrect:
Gland
Reason
Suprarenal (Adrenal gland)
Contains cortex and medulla but no follicles or colloid .
Pancreas
Contains islets of Langerhans and acini , not follicles with colloid.
Parathyroid
Contains chief cells and oxyphil cells , but no colloid or follicles .
Adrenal
Another term for suprarenal gland , again no colloid present.
Summary:
The thyroid gland is the only gland with follicles containing colloid , making it histologically unique among endocrine organs.
Think about the primary function of the anterior pituitary (adenohypophysis) : it secretes trophic hormones that regulate other endocrine glands. Does it contain cells that produce pigment or control skin color , or is that function located elsewhere?
37 / 109
Category:
Endo – Histology
Which of the following cells are not present in the anterior pituitary?
The anterior pituitary (adenohypophysis) is made up of specialized hormone-secreting cells , each type producing specific tropic hormones that regulate various body functions.
Major Cell Types in the Anterior Pituitary:
Cell Type
Hormone Secreted
Gonadotrophs
LH and FSH (reproductive hormones)
Thyrotrophs
TSH (thyroid-stimulating hormone)
Lactotrophs
Prolactin (milk production)
Somatotropes
Growth hormone (GH)
Corticotrophs
ACTH (adrenal stimulation)
Why Melanocytes Are the Correct Answer (Not Present in Anterior Pituitary):
Melanocytes are pigment-producing cells found in the skin, eyes, and hair follicles , where they produce melanin .
They are NOT present in the anterior pituitary .
Melanocyte-stimulating hormone (MSH) is related to pro-opiomelanocortin (POMC) cleavage , but MSH production primarily occurs in the intermediate lobe of the pituitary in some animals, and minimally in humans .
Why the Other Options Are Incorrect (They ARE present in the anterior pituitary):
Cell Type
Reason
Gonadotrophs
✅ Present. They produce LH and FSH .
Thyrotrophs
✅ Present. They produce TSH .
Lactotrophs
✅ Present. They produce Prolactin .
Somatotropes
✅ Present. They produce Growth Hormone (GH) .
Summary:
Melanocytes are not found in the anterior pituitary. They are located in the skin and other peripheral tissues , not in endocrine organs.
Consider the sequence of steps in thyroid hormone synthesis . Iodine must first be actively transported into the thyroid cell before it can be oxidized or organified . Which enzyme comes later in the process , and therefore is not involved in the actual trapping of iodine into the cell?
38 / 109
Category:
Endo – Physio
Iodine trapping does not depend upon:
Mechanism:
Why Peroxidase is Not Involved in Trapping:
Thyroid peroxidase (TPO) is involved in the next steps , not in trapping:
Oxidation of iodide (I⁻ to I₂)
Iodination of tyrosine residues on thyroglobulin (organification)
Coupling of iodotyrosines to form T3 and T4
Since iodine trapping happens before TPO acts , peroxidase is not required for iodine trapping .
Why the Other Options Are Involved in Iodine Trapping:
Option
Role in Iodine Trapping
Ca++
Calcium is involved in thyroid-stimulating hormone (TSH) signal transduction , which stimulates the NIS and enhances iodide uptake.
ATP
Energy is required to maintain the sodium gradient via the Na⁺/K⁺ ATPase , indirectly driving iodine uptake.
Sodium co-transport
The sodium-iodide symporter (NIS) uses sodium co-transport to bring iodide into the cell.
Thyrocytes
Thyroid follicular cells (thyrocytes) are the actual cells that perform iodine trapping .
Summary:
Peroxidase is not required for iodine trapping because it functions in iodide oxidation and organification, not in the initial transport of iodide into the thyroid cell.
Think about how dietary carbohydrates are processed before glucose can be absorbed into the bloodstream . What kind of enzyme works in the small intestine to break down complex sugars into absorbable monosaccharides ?
39 / 109
Category:
Endo – Biochemistry
Which of the following is involved in the absorption of glucose from the intestine?
The absorption of glucose from the intestine requires the breakdown of complex carbohydrates (like starch and disaccharides) into monosaccharides (glucose, galactose, fructose) .
Role of Alpha-Glucosidase:
Alpha-glucosidase is an enzyme located on the brush border of the small intestine (enterocytes) .
It breaks down oligosaccharides and disaccharides into glucose , which can then be absorbed by intestinal transporters (like SGLT1 ).
Without this step, glucose cannot be absorbed , even if large carbohydrate molecules are present.
Clinical Relevance:
Alpha-glucosidase inhibitors (e.g., acarbose, miglitol ) are used to treat diabetes mellitus because they slow carbohydrate digestion , reducing postprandial blood glucose spikes.
Why the Other Options Are Incorrect:
Option
Explanation
Acetyl-CoA carboxylase
This is a rate-limiting enzyme in fatty acid synthesis , not involved in carbohydrate digestion or glucose absorption.
Xanthine oxidase
This enzyme is involved in purine metabolism , converting hypoxanthine to xanthine and then to uric acid . It has no role in glucose absorption.
Carnitine palmitoyl transferase
This enzyme is important in fatty acid oxidation , transporting long-chain fatty acids into mitochondria . It’s unrelated to glucose absorption.
Tyrosine kinase
Tyrosine kinases are intracellular signaling enzymes that mediate hormone actions (like insulin), but they do not digest carbohydrates or absorb glucose in the intestine .
Summary:
Alpha-glucosidase is the enzyme responsible for breaking down carbohydrates into glucose for absorption in the intestine , making it the correct answer.
Think about which hormone is essential for survival during physical stress . Without it, the body cannot maintain blood pressure, glucose levels, or a proper response to illness , which can lead to shock and death if not treated.
40 / 109
Category:
Endo – Physio
Which hormone deficiency causes life-threatening conditions?
While deficiencies in many hormones can cause serious problems, cortisol deficiency is the most immediately life-threatening because cortisol is critical for survival , especially in times of stress.
Functions of Cortisol:
Maintains blood pressure (supports vascular tone and responsiveness to catecholamines)
Regulates glucose metabolism (increases gluconeogenesis and prevents hypoglycemia during fasting or stress)
Suppresses excessive inflammation
Supports cardiovascular function during stress
Why cortisol deficiency is life-threatening:
In Addison’s disease (primary adrenal insufficiency) , lack of cortisol can lead to an Addisonian crisis , characterized by:
Severe hypotension or shock
Hypoglycemia
Hyponatremia and hyperkalemia (due to concurrent aldosterone deficiency in primary cases)
Vomiting, abdominal pain, and weakness
Without immediate corticosteroid replacement , this condition is often fatal .
Why the Other Options Are Incorrect (Less Immediate Risk of Death from Deficiency):
Option
Why it’s less likely to cause immediate death
Insulin
Insulin deficiency causes diabetes mellitus , specifically type 1 diabetes , which is serious but not instantly fatal unless it leads to diabetic ketoacidosis (DKA) , which takes time to develop.
Thyroxine
Thyroxine (T4) deficiency causes hypothyroidism , which can be life-threatening if severe (myxedema coma ), but typically develops slowly over time , not acutely.
Aldosterone
Aldosterone deficiency can cause electrolyte disturbances , but it is usually part of primary adrenal insufficiency , where cortisol deficiency is the main life-threatening factor .
Glucagon
Glucagon deficiency might cause hypoglycemia , but in most people, other mechanisms compensate for low glucagon. It is not typically life-threatening on its own.
Summary:
Cortisol is the hormone whose deficiency most rapidly leads to life-threatening conditions , especially during stress, illness, or injury.
Consider the timing of the hemorrhage in relation to childbirth. Does Sheehan syndrome typically occur before delivery , or is it more commonly related to events that happen during or after childbirth ?
41 / 109
Category:
Endo – Pathology
Which of the following is wrong regarding Sheehan syndrome?
Sheehan syndrome is a condition resulting from ischemic necrosis of the anterior pituitary , typically following severe postpartum hemorrhage (PPH) .
Pathophysiology:
During pregnancy, the anterior pituitary enlarges , mainly due to increased prolactin-secreting cells to prepare for lactation .
Despite the enlargement, the blood supply does not increase proportionally , making the pituitary vulnerable to ischemia .
If a woman experiences massive postpartum hemorrhage , the sudden drop in blood pressure can cause hypoperfusion of the anterior pituitary , leading to necrosis and hypopituitarism .
Key Clinical Features:
Failure to lactate (due to prolactin deficiency)
Amenorrhea or oligomenorrhea (due to gonadotropin deficiency)
Hypothyroidism and adrenal insufficiency may also develop over time due to TSH and ACTH deficiencies .
Why the Other Options Are Correct (True Statements):
Option
Explanation
It is a result of ischemic necrosis of pituitary
✅ True. This is the fundamental cause of Sheehan syndrome.
It is associated with infarction of the anterior lobe
✅ True. The anterior pituitary is mainly affected; the posterior pituitary is usually spared because it has a direct arterial blood supply.
It is associated with postpartum hemorrhage
✅ True. Massive postpartum hemorrhage is the classic trigger .
During pregnancy anterior pituitary enlarges to twice its size
✅ True. The anterior pituitary enlarges, mostly due to prolactin cell hyperplasia , increasing its vulnerability to ischemia.
Why “It is associated with antepartum hemorrhage” is Incorrect:
Antepartum hemorrhage occurs before delivery (e.g., placenta previa, placental abruption).
**Sheehan syndrome is specifically associated with postpartum hemorrhage , meaning bleeding after delivery .
Therefore, the statement about antepartum hemorrhage is wrong .
Think about the location of the thyroid gland in the neck relative to the larynx and trachea . Which vertebral levels correspond to the region where the thyroid cartilage, cricoid cartilage, and upper tracheal rings are located?
42 / 109
Category:
Endo – Anatomy
The thyroid gland is at the level of which of the following?
The thyroid gland is located in the anterior neck , specifically:
Vertebral Levels:
The thyroid gland typically spans from C5 to T1 vertebral levels.
The isthmus usually lies over the 2nd to 4th tracheal rings , while the lateral lobes extend upwards and downwards along the trachea and larynx.
Anatomic Landmarks:
Structure
Level
Thyroid cartilage
C4-C5
Cricoid cartilage
C6
Thyroid gland
C5–T1
Why the Other Options Are Incorrect:
C6–T1
C8–T2
The cervical spine only goes up to C7 , so C8 is not a vertebral level (C8 refers to a spinal nerve root, not a vertebra).
This option is anatomically incorrect.
C4–C8
C8 again is not a vertebra .
Additionally, the thyroid typically does not extend up to C4 unless it’s enlarged.
T1–T3
Think about the primary function of the adrenal medulla in the fight or flight response . Which hormone causes rapid heart rate, increased blood flow to muscles, bronchodilation, and energy mobilization during acute stress?
43 / 109
Category:
Endo – Physio
80% of secretion of the adrenal medulla consists of which of the following?
The adrenal medulla is the inner part of the adrenal gland , and it functions as a neuroendocrine organ . It secretes catecholamines in response to sympathetic stimulation , especially during stress .
Main Secretory Product: Adrenaline (Epinephrine)
Approximately 80% of adrenal medulla secretion is adrenaline (epinephrine).
The remaining 20% is mostly norepinephrine , with very small amounts of dopamine.
Why does the adrenal medulla secrete mostly adrenaline?
The adrenal medulla contains the enzyme phenylethanolamine-N-methyltransferase (PNMT) , which converts norepinephrine into epinephrine .
This conversion is stimulated by cortisol coming from the adrenal cortex —this is why the adrenal medulla produces so much epinephrine relative to norepinephrine.
Functions of Adrenaline:
Increases heart rate (positive chronotropy)
Increases cardiac output (positive inotropy)
Dilates bronchioles
Mobilizes energy stores (glycogenolysis, lipolysis)
Prepares the body for “fight or flight”
Why the Other Options Are Incorrect:
Cortisol
Cortisol is secreted by the adrenal cortex , not the medulla.
It plays a role in metabolism and stress response , but it is not part of adrenal medulla secretion .
Adrenocorticotropic hormone (ACTH)
ACTH is secreted by the anterior pituitary , not the adrenal medulla.
ACTH stimulates the adrenal cortex , not the medulla.
Aldosterone
Aldosterone is a mineralocorticoid secreted by the zona glomerulosa of the adrenal cortex .
It regulates sodium and potassium balance , not part of adrenal medulla output.
Norepinephrine
The adrenal medulla does secrete norepinephrine , but it makes up only about 20% of its catecholamine output .
80% is adrenaline (epinephrine).
Consider how the body normally controls cortisol production through negative feedback . In a healthy person, giving an external source of corticosteroids should suppress their own cortisol production. If this feedback loop is broken, what does that tell you about the underlying problem?
44 / 109
Category:
Endo – Physio
Best diagnostic test for Cushing syndrome:
Cushing syndrome refers to excess cortisol production , which can result from:
Best initial diagnostic test: The Dexamethasone Suppression Test
Dexamethasone is a synthetic glucocorticoid that normally suppresses ACTH production via negative feedback to the pituitary.
In normal individuals , a low-dose dexamethasone test lowers cortisol levels .
In Cushing syndrome , cortisol secretion is not suppressed due to:
Why is it the best test?
The low-dose dexamethasone suppression test is recommended by most guidelines (e.g., Endocrine Society ) as the first-line screening test for suspected Cushing syndrome.
It helps distinguish between physiologic causes of high cortisol (stress, obesity, etc.) and true Cushing syndrome .
Why the Other Options Are Incorrect:
ACTH levels
Serum cortisol levels
Single cortisol measurements are not reliable because cortisol has a diurnal rhythm (high in the morning, low at night).
A random cortisol level can be misleading and does not diagnose Cushing syndrome.
Blood glucose levels
Hyperglycemia is common in Cushing syndrome due to cortisol-induced insulin resistance.
However, blood glucose testing is not diagnostic of Cushing syndrome—it just detects a complication .
24-hour urine collection for free cortisol
Test
Purpose
Practical Use
Low-dose dexamethasone suppression test (DST)
First-line screening test (most practical and commonly recommended first step)
✅
24-hour urinary free cortisol (UFC)
Alternative first-line test but often used for confirmation or additional screening
✅ but less practical
Late-night salivary cortisol
Also first-line, but less commonly chosen for initial workup in exams
✅
Consider the developmental origin of endocrine glands during fetal life. If a hormone-producing organ never forms properly, would supplementation after birth be enough to prevent symptoms, or would the condition already be present from birth?
45 / 109
Category:
Endo – Embryology
What is the most common cause of congenital hypothyroidism?
Congenital hypothyroidism is a condition where the newborn has low thyroid hormone levels at birth . It is one of the most common preventable causes of mental retardation if not treated early, which is why newborn screening programs universally test for it .
Most Common Cause: Thyroid Dysgenesis
Thyroid dysgenesis refers to the failure of the thyroid gland to develop properly during fetal life. This can involve:
Aplasia (complete absence of the thyroid)
Hypoplasia (underdeveloped thyroid)
Ectopic thyroid tissue (thyroid tissue located in the wrong place, often along the path of embryonic descent)
This accounts for about 85-90% of congenital hypothyroidism cases , making it the most common cause .
Why the Other Options Are Incorrect:
Hereditary
Obesity
Obesity is not a cause of congenital hypothyroidism .
While thyroid problems can affect metabolism, obesity is not related to congenital cases .
Iodine deficiency
Iodine deficiency is a global cause of hypothyroidism , but in developed countries , it is not the most common cause of congenital hypothyroidism due to iodine supplementation in salt and prenatal care.
In certain endemic regions , iodine deficiency can cause goitrous hypothyroidism in neonates, but thyroid dysgenesis remains more common overall .
Lack of iodine intake in childhood
When a patient presents with low blood pressure and blood sugar instability , consider which hormone from the adrenal cortex is essential for maintaining both vascular tone and metabolic balance . What is the body’s primary hormonal defense in times of physical or emotional stress?
46 / 109
Category:
Endo – Physio
A lean lady comes to the clinic with the complaint of fragility and irritability. On examination, her blood pressure is 90/60 mmHg. Her laboratory results show blood sugar 240 mg/dL. What test would you first suggest to be done?
In this case, the patient is a lean lady with hypotension (90/60 mmHg), irritability, and hyperglycemia (240 mg/dL) . These findings suggest an underlying endocrine problem , most likely involving the adrenal glands .
Why test cortisol first?
Cortisol is the major stress hormone produced by the adrenal cortex .
It plays a crucial role in:
Maintaining blood pressure by enhancing the responsiveness of blood vessels to catecholamines (like adrenaline).
Regulating glucose metabolism , typically raising blood glucose levels during fasting or stress.
Deficiency of cortisol (adrenal insufficiency ) leads to:
Hypotension due to lack of vascular tone support
Blood sugar dysregulation, though typically hypoglycemia is seen, some atypical cases may present with stress-induced hyperglycemia , especially if there is underlying glucose intolerance or diabetes.
Testing serum cortisol (preferably morning cortisol) is the first diagnostic step when adrenal insufficiency is suspected.
After cortisol measurement, further tests like ACTH levels or stimulation tests may be performed to pinpoint the cause.
Why the Other Options Are Incorrect:
Electrolytes
While electrolyte testing (checking for hyponatremia, hyperkalemia) can support adrenal insufficiency diagnosis, it is not the first step .
Electrolyte changes are secondary findings , whereas cortisol directly addresses the primary issue .
T3
Triiodothyronine (T3) measures thyroid function.
Although thyroid dysfunction can cause irritability or fatigue, it is not the main concern here , especially given the presence of hypotension and hyperglycemia .
ACTH
T4
Think about the solubility and chemical nature of steroid hormones. Since they are lipid-based , consider which cellular organelle specializes in handling lipid metabolism and synthesis , rather than protein production.
47 / 109
Category:
Endo – Physio
Where are the steroid hormones synthesized in the cell?
Steroid hormones are synthesized from cholesterol , making them lipid-derived hormones . Examples include:
Glucocorticoids (e.g., cortisol)
Mineralocorticoids (e.g., aldosterone)
Sex hormones (e.g., estrogen, testosterone, progesterone)
Site of synthesis: Smooth Endoplasmic Reticulum (SER)
The smooth endoplasmic reticulum is the primary site for steroid hormone synthesis in steroidogenic cells (e.g., adrenal cortex, gonads).
The mitochondria also participate in the initial and final steps of steroidogenesis, but the bulk of the enzymatic modifications happen in the SER .
Enzymes such as cytochrome P450 family members located in the SER membranes catalyze the conversion of cholesterol into various steroid hormones.
Why the Other Options Are Incorrect:
Cytoplasm
Steroid hormones are not synthesized in the free cytoplasm .
However, cholesterol transport to organelles starts in the cytoplasm, but the enzymatic steps happen in the SER and mitochondria .
Nucleus
The nucleus is involved in gene transcription but does not synthesize hormones .
Steroid hormones act on nuclear receptors , but they are not made in the nucleus .
Golgi bodies
The Golgi apparatus is mainly involved in protein processing, modification, and packaging .
It is not involved in steroid synthesis , though it processes some peptide hormones.
Rough endoplasmic reticulum (RER)
The RER is studded with ribosomes and is specialized for protein synthesis , not lipid or steroid metabolism.
Steroidogenesis occurs in the SER, not the RER .
When the body is in a fasting state, it needs to maintain blood glucose levels. Consider what stored forms of energy the liver can mobilize quickly to increase glucose availability. Which process breaks down these stored reserves?
48 / 109
Category:
Endo – Physio
Which of the following does glucagon enhance?
Glucagon is a catabolic hormone secreted by the alpha cells of the pancreas in response to low blood glucose levels (hypoglycemia) . Its primary target is the liver , where it acts to increase blood glucose levels .
Main Action: Hepatic Glycogenolysis
Glycogenolysis is the breakdown of glycogen into glucose .
In the liver, glucagon activates glycogen phosphorylase , the key enzyme that liberates glucose-1-phosphate from glycogen , ultimately producing free glucose that enters the bloodstream.
This process is crucial during fasting , exercise , or hypoglycemia to maintain adequate blood glucose levels.
Additional actions of glucagon:
Stimulates gluconeogenesis (making new glucose from non-carbohydrate sources like amino acids)
Promotes lipolysis indirectly , providing energy substrates for the liver
Inhibits glycogenesis and protein synthesis in the liver
Why the Other Options Are Incorrect:
Amino acid uptake
Glucagon does increase the availability of amino acids for gluconeogenesis , but it generally does not enhance cellular amino acid uptake for anabolic purposes .
Instead, it stimulates the liver to convert amino acids into glucose , not to store them.
Protein formation
Lipid oxidation
While glucagon can indirectly promote lipolysis in adipose tissue , the primary hormone for lipid oxidation is epinephrine .
Glucagon does not directly stimulate lipid oxidation in the liver or muscle as its main function.
Hepatic glycogenesis
Glycogenesis is the formation of glycogen from glucose , which is promoted by insulin , not glucagon.
Glucagon inhibits glycogenesis to prevent glucose storage during fasting.
Consider where in the body carbohydrates are broken down before they can be absorbed. Think about enzymes involved in the final steps of carbohydrate digestion. How might blocking these enzymes help control post-meal blood sugar spikes?
49 / 109
Category:
Endo – Pharmacology
Which drug inhibits glucose absorption in the small intestine?
Glucosidase inhibitors are a class of oral antidiabetic drugs that specifically target the small intestine to inhibit carbohydrate absorption .
How do they work?
Alpha-glucosidase enzymes , located in the brush border of the small intestine , are responsible for breaking down complex carbohydrates (starches, disaccharides) into simple sugars (glucose) for absorption.
Glucosidase inhibitors (e.g., Acarbose , Miglitol ) block these enzymes , delaying carbohydrate digestion.
As a result, glucose absorption is slowed down , leading to a blunted postprandial (after-meal) rise in blood sugar levels .
This mechanism is especially helpful in managing postprandial hyperglycemia in type 2 diabetes mellitus .
Clinical effect:
Decreases post-meal blood glucose peaks
Minimal risk of hypoglycemia when used alone
Common side effects: flatulence, bloating, and diarrhea (due to undigested carbs fermenting in the colon)
Why the Other Options Are Incorrect:
Metformin
Metformin is a biguanide that works by:
It does not inhibit intestinal glucose absorption directly , although it may mildly reduce glucose uptake from the gut in some cases—but this is not its primary mechanism .
Insulin aspart
Rosiglitazone
Rosiglitazone is a thiazolidinedione (TZD) .
It works by activating PPAR-γ receptors , which increase insulin sensitivity in adipose tissue and muscle.
It has no effect on glucose absorption in the intestine .
Exogenous glucose
Consider the role of somatostatin as a global inhibitor of hormone secretion. In the pancreas, think about the balance between the hormones that raise and lower blood glucose. Why might the body benefit from temporarily halting both actions simultaneously during certain conditions?
50 / 109
Category:
Endo – Physio
Somatostatin decreases which of the following?
Somatostatin is known as the “universal inhibitor” because it suppresses the secretion of multiple hormones across different organ systems.
In the pancreas , somatostatin is secreted by the delta cells of the islets of Langerhans , where it inhibits both insulin and glucagon secretion:
Insulin (from beta cells) lowers blood glucose by promoting cellular uptake of glucose.
Glucagon (from alpha cells) raises blood glucose by stimulating glycogenolysis and gluconeogenesis.
Somatostatin regulates this system by inhibiting both , preventing rapid swings in glucose levels and providing a brake mechanism on pancreatic activity.
Why does this happen?
Somatostatin prevents overstimulation of both anabolic and catabolic pathways.
It promotes homeostasis during digestion by slowing nutrient absorption and limiting hormone release, ensuring that blood glucose levels don’t fluctuate wildly.
Somatostatin also inhibits growth hormone (GH), thyroid-stimulating hormone (TSH), and various gastrointestinal hormones , but in the pancreatic context , its key role is to reduce both insulin and glucagon secretion.
Why the Other Options Are Incorrect:
Cortisol only
Cortisol and Insulin
Again, somatostatin inhibits insulin , but it does not directly inhibit cortisol .
Cortisol secretion is under the control of the HPA axis , not the pancreas.
Insulin only
Somatostatin inhibits both insulin and glucagon , not insulin alone.
This option is partially correct but incomplete .
Glucagon only
Consider the role of this hormone in rapidly modulating the release of other hormones across different organs. Would a hormone that acts quickly and degrades fast be expected to have a large or small amino acid chain?
51 / 109
Category:
Endo – Physio
What are the number of amino acids in somatostatin?
Somatostatin is a peptide hormone known for its inhibitory effects on various endocrine and exocrine functions. It was originally discovered in the hypothalamus, where it inhibits the release of growth hormone , but it is also secreted in other areas such as the pancreas and gastrointestinal tract.
Number of Amino Acids:
Somatostatin exists in two biologically active forms :
The predominant and classic form is somatostatin-14 , especially in the hypothalamus and anterior pituitary .
It acts quickly and has a short half-life , making a short peptide structure ideal for its regulatory functions.
Functions of Somatostatin:
Inhibits growth hormone release (from the anterior pituitary)
Inhibits TSH , insulin , glucagon , and gastric secretions
Acts as a universal inhibitor in endocrine physiology
Its short peptide structure (14 amino acids) supports its rapid action and degradation , which is ideal for a hormone that must finely regulate other hormone secretions moment-to-moment.
Why the Other Options Are Incorrect:
10
151
This refers to calcitonin gene-related peptide (CGRP) or sometimes larger proteins , not somatostatin.
Somatostatin is much shorter and does not have this many amino acids.
89
51
This is the number of amino acids in insulin , which is made of two chains (A and B) linked by disulfide bonds.
Somatostatin, by contrast, is a simple, single-chain peptide .
Think about the size and structure of hormones. Small hormones like insulin are peptides, while larger ones like growth hormone are considered protein hormones . Consider the biological actions of growth hormone—would a small or large polypeptide be required to mediate such widespread effects?
52 / 109
Category:
Endo – Physio
What are the number of amino acids in growth hormone?
Growth Hormone (GH) , also known as somatotropin , is a single-chain polypeptide hormone produced by the anterior pituitary gland . It plays a central role in growth, metabolism, and development , influencing tissues throughout the body.
Number of Amino Acids:
**Growth hormone is composed of exactly 191 amino acids .
Its molecular weight is approximately 22 kDa (kilodaltons) .
GH acts by binding to growth hormone receptors (GHR) on target cells, initiating processes such as:
Why is the number of amino acids important?
The size of the hormone determines how it interacts with its receptor, its half-life , and how it is synthesized, stored, and secreted .
Knowing the amino acid composition is also relevant in biotechnology , where recombinant growth hormone (rhGH) is produced for medical use, and the 191 amino acid sequence must be precisely replicated .
Why the Other Options Are Incorrect:
10
A hormone with only 10 amino acids would be an oligopeptide , not a protein.
Examples include vasopressin (9 amino acids) and oxytocin (9 amino acids) —not growth hormone.
51
This is the number of amino acids in insulin , not growth hormone.
Insulin has two chains (A and B chains) connected by disulfide bonds .
230
There is no common human hormone with 230 amino acids involved in growth regulation.
GH has 191 amino acids , and though pre-hormone forms (preprohormones) can be longer, mature GH is 191 amino acids long .
89
This is the approximate size of parathyroid hormone (PTH) , which has 84 amino acids .
PTH regulates calcium and phosphate metabolism , not growth.
When the body perceives a threat, it doesn’t just activate the nervous system—it also triggers a coordinated immune response. Think about which molecules serve as signals between the immune system and the brain , initiating changes in behavior, metabolism, and hormonal pathways during stress.
53 / 109
Category:
Endo – Physio
Which one of the following is a mediator in stress?
Stress activates a complex network of physiological responses involving the nervous, endocrine, and immune systems . One of the key immune mediators that plays a role in the stress response is Interleukin 1 (IL-1) .
How does IL-1 mediate stress?
IL-1 is a pro-inflammatory cytokine , produced primarily by activated macrophages .
It serves as a bridge between the immune system and the hypothalamic-pituitary-adrenal (HPA) axis during stress.
IL-1 stimulates the hypothalamus to release corticotropin-releasing hormone (CRH) , which in turn activates the release of ACTH from the pituitary, leading to cortisol secretion from the adrenal cortex.
Cortisol then exerts effects on glucose metabolism , immune suppression , and helps the body cope with stress.
Additional Role of IL-1 in Stress:
It induces fever , fatigue , and behavioral changes commonly seen during illness or prolonged stress.
It is also involved in promoting sickness behavior , which conserves energy during immune activation.
Thus, IL-1 is not just an immune molecule—it actively participates in the body’s stress signaling network.
Why the Other Options Are Incorrect:
Interleukin 4 (IL-4)
IL-4 is primarily involved in allergic reactions and humoral immunity .
It stimulates B cell proliferation and IgE class switching , but it does not mediate the stress response .
Interleukin 2 (IL-2)
IL-2 promotes the growth and differentiation of T-cells .
While important for immune system regulation , it does not have a central role in stress signaling or HPA axis activation .
Interleukin 3 (IL-3)
IL-3 is involved in hematopoiesis , helping to stimulate the growth of blood cell precursors in the bone marrow.
It is not a mediator of the stress response .
Interferon gamma (IFN-γ)
IFN-γ is a key player in cell-mediated immunity and macrophage activation , especially in defense against intracellular pathogens .
While it participates in immune regulation, it does not directly mediate stress-related signaling to the brain or HPA axis .
Think adrenal destruction due to chronic infection.
54 / 109
Category:
Endo – Pathology
A person with pulmonary tuberculosis undergoing treatment for 3 months is likely to develop which pathology?
Pulmonary tuberculosis (TB) can spread to the adrenal glands via hematogenous routes, especially in long-standing or poorly managed cases. Over time, this can destroy adrenal tissue , leading to primary adrenal insufficiency , also known as Addison disease . Patients may develop symptoms like hypotension, fatigue, hyponatremia, and hyperpigmentation due to increased ACTH production. TB used to be a leading cause of Addison disease worldwide, especially in developing countries.
Answer breakdown:
✅ Addison disease – Correct; due to TB destroying adrenal cortex
Cushing syndrome – Incorrect; excess cortisol, not linked to TB
Conn syndrome – Incorrect; primary hyperaldosteronism
Hypertension – Incorrect; Addison disease causes hypo tension
Cushing disease – Incorrect; pituitary ACTH excess, not related to TB
This hormone is derived from the same precursor as MSH (melanocyte-stimulating hormone).
55 / 109
Category:
Endo – Physio
A person may get brown skin due to which of the following conditions?
ACTH (Adrenocorticotropic Hormone) is derived from POMC (proopiomelanocortin) , which also gives rise to MSH (melanocyte-stimulating hormone) . When ACTH levels rise—such as in Addison’s disease or primary adrenal insufficiency —it indirectly increases melanin synthesis , especially in the buccal mucosa and skin creases , leading to hyperpigmentation . This is because the body tries to compensate for low cortisol by ramping up ACTH production, which incidentally increases pigmentation too.
Answer breakdown:
Increased activity of somatotrophs – Incorrect (leads to increased GH, not related to pigmentation)
Decreased ACTH production – Incorrect (would reduce pigmentation)
Decreased melanin – Incorrect (would cause paler skin)
✅ Increased ACTH production – Correct
Decreased activity of somatotrophs – Incorrect (irrelevant to melanin production)
This glucocorticoid is secreted from the middle and inner layers, but not from the outermost zone of the cortex.
56 / 109
Category:
Endo – Physio
What zone(s) of the adrenal gland secrete cortisol?
The adrenal cortex is divided into three zones:
Zona glomerulosa (outermost) — secretes mineralocorticoids like aldosterone.
Zona fasciculata (middle) — the main site for glucocorticoid production, particularly cortisol .
Zona reticularis (innermost) — secretes androgens and also contributes to some cortisol secretion.
Thus, cortisol is primarily produced by the zona fasciculata and to a lesser extent by the zona reticularis .
Answer breakdown:
Zona glomerulosa – Incorrect (produces aldosterone)
Zona fasciculata and reticularis – ✅ Correct
Zona fasciculata – Partially correct (major contributor but not sole)
Zona reticularis – Partially correct (minor contributor)
Zona pellucida – Incorrect (not a real zone of the adrenal cortex)
The anterior pituitary (adenohypophysis) does not receive its blood supply directly from a major artery like the posterior pituitary does. Instead, it gets hormones from the hypothalamus through a special two-capillary system . Think about which artery connects with the hypothalamus first to allow this.
57 / 109
Category:
Endo – Anatomy
Which of the following statements is incorrect regarding the hypothalamic-hypophyseal portal system?
The hypothalamic-hypophyseal portal system is a specialized vascular network that allows hypothalamic hormones (like TRH, CRH, GnRH, etc.) to travel directly to the anterior pituitary (adenohypophysis) . It originates from the superior hypophyseal artery , a branch of the internal carotid artery. These arteries form a capillary plexus in the median eminence and infundibulum , which drain into portal veins leading to a secondary plexus in the anterior pituitary.
In contrast, the inferior hypophyseal artery supplies the posterior pituitary and does not contribute to the portal system, making that statement incorrect .
Answer Breakdown:
It arises from the superior hypophyseal artery ✅ (True)
It arises from the inferior hypophyseal artery ❌ (False – incorrect, hence the correct choice)
Capillary plexus in infundibulum ✅ (True)
Transports releasing/inhibiting factors ✅ (True)
None of these ❌ (There is an incorrect statement)
Think aldosterone’s effect on sodium and water retention.
58 / 109
Category:
Endo – Physio
An excess of mineralocorticoids will cause which of the following?
Mineralocorticoids, especially aldosterone , act on the distal tubules and collecting ducts of the kidney to promote sodium reabsorption and potassium excretion . Water follows sodium, so reabsorption leads to increased extracellular fluid volume and ultimately increased blood pressure . This is seen in conditions like primary hyperaldosteronism (Conn’s syndrome) , where patients present with hypertension , hypokalemia , and metabolic alkalosis .
Other options are incorrect:
High cortisol: Glucocorticoid excess, not mineralocorticoid.
High GH: Unrelated to mineralocorticoid activity.
Low blood pressure: The opposite effect of mineralocorticoid action.
Low blood osmolarity: Aldosterone does not significantly dilute plasma since water and sodium are reabsorbed proportionally.
Answer Breakdown:
High cortisol ❌ (Glucocorticoid-related)
High growth hormone ❌ (Pituitary hormone, not involved here)
Low blood pressure ❌ (Opposite effect)
Low osmolarity of blood ❌ (Water follows Na⁺, maintaining osmolarity)
High blood pressure ✅ (Correct – due to Na⁺ and water retention)
This is the outermost layer of the adrenal cortex, primarily influenced by the renin-angiotensin system rather than ACTH.
59 / 109
Category:
Endo – Physio
What part of the adrenal cortex secretes aldosterone?
The adrenal cortex has three layers, each secreting a different class of steroid hormones:
Zona glomerulosa (outermost): Secretes mineralocorticoids , primarily aldosterone , which regulates sodium and potassium balance and blood pressure. Its secretion is mostly controlled by the renin-angiotensin-aldosterone system (RAAS) and serum potassium levels , not ACTH.
Zona fasciculata (middle): Secretes glucocorticoids , mainly cortisol , involved in metabolism and stress response.
Zona reticularis (innermost): Secretes androgens (e.g., DHEA), important for secondary sexual characteristics.
Answer Breakdown:
Zona fasciculata and reticularis ❌ (Secrete cortisol and androgens, not aldosterone)
Zona reticularis ❌ (Secretes androgens)
Zona fasciculata ❌ (Secretes cortisol)
All of these ❌ (Only zona glomerulosa secretes aldosterone)
Zona glomerulosa ✅ (Correct site of aldosterone production)
This hormone is secreted by somatotropes , which are the most abundant cells in the anterior pituitary.
60 / 109
Category:
Endo – Physio
What is the correct match-up between the hormone and its percentage secretion from the adenohypophysis?
The anterior pituitary (adenohypophysis) secretes multiple hormones, each from a specific cell type. Among these, somatotropes make up about 50% of the anterior pituitary cell population , making growth hormone (GH) the most abundantly secreted hormone. GH plays a critical role in stimulating growth, cell reproduction, and regeneration, especially during childhood and adolescence.
Other hormones like ACTH, prolactin, LH, and FSH are secreted in much smaller proportions. Prolactin-producing cells (lactotropes) account for about 10–20%, while gonadotropes (FSH, LH) and corticotropes (ACTH) are even fewer.
Answer Breakdown:
Adrenocorticotropic hormone – 40% ❌ (Too high; corticotropes are fewer)
Prolactin – 40% ❌ (Overestimated; lactotropes are 10–20%)
Follicle-stimulating hormone – 20% ❌ (Gonadotropes produce both FSH & LH, together <20%)
Leutinizing hormone – 50% ❌ (Again, gonadotropes are not that abundant)
Growth hormone – 50% ✅ (Somatotropes are the most numerous pituitary cells)
Insulin is a peptide hormone that uses a membrane-bound receptor with intrinsic enzymatic activity
61 / 109
Category:
Endo – Physio
What is the mechanism of action of insulin on the target cell?
Insulin acts via a tyrosine kinase receptor located on the cell membrane. When insulin binds, the receptor undergoes autophosphorylation , activating downstream signaling pathways such as PI3K-Akt and MAPK . These pathways increase glucose uptake (via GLUT4 translocation), glycogen synthesis, lipid synthesis, and protein synthesis.
Unlike many hormones that act through second messengers like cAMP or cGMP , insulin’s receptor itself functions as an enzyme , making its action direct and rapid .
Answer Breakdown:
cAMP → Used by glucagon and epinephrine
Intracellular → Used by steroid hormones
Tyrosine kinase ✅ → Used by insulin
cGMP → Used by nitric oxide and ANP
Phospholipase C → Used by hormones like oxytocin and vasopressin (via IP3/DAG pathway)
This syndrome results from excess cortisol — think of what cortisol increases and stores .
62 / 109
Category:
Endo – Pathology
Which of the following is not seen in Cushing’s syndrome?
Cushing’s syndrome is caused by prolonged exposure to high levels of cortisol , whether endogenous (e.g. ACTH-producing tumor) or exogenous (e.g. long-term steroid therapy). Cortisol promotes gluconeogenesis, lipogenesis, and protein breakdown , leading to classic features like:
Central obesity and weight gain
Hyperglycemia and insulin resistance
Muscle wasting and thin limbs
Purple abdominal striae from skin thinning
Hirsutism due to excess adrenal androgensWeight loss is not typical in Cushing’s — the opposite occurs due to fat accumulation and fluid retention.
Answer Breakdown:
Hirsutism → Common due to androgen excess
Weight gain → Classic symptom
Hyperglycemia → Due to cortisol’s metabolic effects
Purple striae → Due to collagen breakdown and skin thinning
Weight loss ✅ → Not seen; thus the correct answer
These cells are located in the thyroid but are not the ones making T3/T4. They deal with calcium.
63 / 109
Category:
Endo – Physio
What type of cell secretes calcitonin?
Calcitonin is secreted by the parafollicular cells (also known as C cells ) of the thyroid gland . These cells are distinct from follicular cells, which produce thyroid hormones (T3 and T4). Calcitonin helps lower blood calcium levels by inhibiting osteoclast activity, thus reducing bone resorption . It’s especially active in children and during periods of bone remodeling.
Answer Breakdown:
Colloid → Refers to the fluid inside thyroid follicles, not a cell type
Corticotrophs → Pituitary cells that release ACTH
Parafollicular cells ✅ → Correct; secrete calcitonin
Pituicytes → Glial cells in the posterior pituitary
Follicular cells → Produce T3 and T4, not calcitonin
These cells are located in the thyroid but are not the ones making T3/T4. They deal with calcium.
64 / 109
Category:
Endo – Physio
What type of cell secretes calcitonin?
Calcitonin is secreted by the parafollicular cells (also known as C cells ) of the thyroid gland . These cells are distinct from follicular cells, which produce thyroid hormones (T3 and T4). Calcitonin helps lower blood calcium levels by inhibiting osteoclast activity, thus reducing bone resorption . It’s especially active in children and during periods of bone remodeling.
Answer Breakdown:
Colloid → Refers to the fluid inside thyroid follicles, not a cell type
Corticotrophs → Pituitary cells that release ACTH
Parafollicular cells ✅ → Correct; secrete calcitonin
Pituicytes → Glial cells in the posterior pituitary
Follicular cells → Produce T3 and T4, not calcitonin
These cells are located in the thyroid but are not the ones making T3/T4. They deal with calcium.
65 / 109
Category:
Endo – Physio
What type of cell secretes calcitonin?
Calcitonin is secreted by the parafollicular cells (also known as C cells ) of the thyroid gland . These cells are distinct from follicular cells, which produce thyroid hormones (T3 and T4). Calcitonin helps lower blood calcium levels by inhibiting osteoclast activity, thus reducing bone resorption . It’s especially active in children and during periods of bone remodeling.
Answer Breakdown:
Colloid → Refers to the fluid inside thyroid follicles, not a cell type
Corticotrophs → Pituitary cells that release ACTH
Parafollicular cells ✅ → Correct; secrete calcitonin
Pituicytes → Glial cells in the posterior pituitary
Follicular cells → Produce T3 and T4, not calcitonin
Think about what PTH does to calcium levels in blood and how it affects bones to achieve that.
66 / 109
Category:
Endo – Physio
Excessive parathyroid hormone causes which of the following conditions?
Parathyroid hormone (PTH) plays a central role in calcium homeostasis . One of its key actions is to increase blood calcium levels by stimulating osteoclast activity , which breaks down bone tissue — a process known as bone resorption . This releases calcium from the bone matrix into the bloodstream. Chronically elevated PTH, as in hyperparathyroidism , leads to excessive bone resorption, causing weakened bones, fractures, and conditions like osteitis fibrosa cystica.
Answer Breakdown:
Bone resorption ✅ → True effect of excess PTH
Collagen degradation → Indirect result but not the main action
Collagen synthesis → Opposite of what PTH promotes
Increased vitamin C uptake → Not regulated by PTH
Bone formation → Opposite of resorption; would occur with calcitonin or GH, not PTH
Think of the anterior pituitary cells that secrete growth hormone — excess of which causes gigantism before epiphyseal plates close.
67 / 109
Category:
Endo – Physio
In gigantism, which of the following cell types is overactive?
Gigantism results from excessive secretion of growth hormone (GH) during childhood, before the epiphyseal growth plates have closed. The hormone responsible is produced by somatotrope cells in the anterior pituitary gland. When these cells become hyperactive or form an adenoma, GH levels rise, leading to abnormal linear bone growth and gigantism. In adults, the same overactivity leads to acromegaly rather than increased height.
Answer Breakdown:
Thyrotropes → secrete TSH, regulate thyroid function
Mammotropes → secrete prolactin, for lactation
Somatotropes → secrete GH → overactivity causes gigantism ✅
Corticotropes → secrete ACTH, stimulate adrenal cortex
Gonadotropes → secrete LH and FSH, for reproduction
Focus on what the anterior pituitary monitors to decide whether to secrete thyrotropin (TSH) — is it checking raw materials for hormone production (like iodine), or is it responding to the final product levels already in the bloodstream?
68 / 109
Category:
Endo – Physio
Which of the following conditions will decrease the rate of secretion of thyrotropin?
Thyrotropin, also known as TSH (Thyroid Stimulating Hormone) , is secreted by the anterior pituitary in response to TRH (thyrotropin-releasing hormone) from the hypothalamus .
TSH stimulates the thyroid gland to release T3 and T4 . These thyroid hormones then exert negative feedback on both the pituitary and hypothalamus :
When thyroid hormone levels are high , the pituitary reduces TSH secretion .
This maintains homeostasis , preventing overproduction of thyroid hormones.
So, high blood levels of T3/T4 decrease TSH secretion.
5) Answer Breakdown:
Concentration of iodine in blood is normal – ❌ No inhibitory effect on TSH.
Concentration of thyroid hormones in blood is low – ❌ This increases TSH via negative feedback.
Concentration of iodine in blood is low – ❌ May increase TSH due to impaired T3/T4 synthesis.
Concentration of iodine in blood is high – ❌ Might transiently reduce T3/T4 but not a primary regulator of TSH.
Concentration of thyroid hormones in blood is high – ✅ Suppresses TSH via negative feedback.
This enzyme activates vitamin D in the kidney. Without it, calcium absorption is impaired → low serum calcium → compensatory rise in PTH.
69 / 109
Category:
Endo – Biochemistry
Secondary hyperparathyroidism can arise due to which of the following?
1α-hydroxylase is the renal enzyme that converts 25-hydroxyvitamin D into its active form: 1,25-dihydroxyvitamin D (calcitriol) .
Calcitriol increases calcium absorption from the intestine. So when 1α-hydroxylase is deficient:
Less calcitriol →
Less calcium absorption from the gut →
Hypocalcemia →
Parathyroid glands respond by secreting more PTH → → Secondary hyperparathyroidism (i.e., PTH increase secondary to hypocalcemia)
This is commonly seen in chronic kidney disease , where 1α-hydroxylase activity is reduced.
5) Answer Breakdown:
21α-hydroxylase deficiency – ❌ Affects adrenal steroid synthesis, not calcium metabolism.
1α-hydroxylase deficiency – ✅ Leads to low calcitriol → hypocalcemia → ↑PTH
Deficiency of both enzymes – ❌ Only 1α-hydroxylase is relevant here
17α-hydroxylase deficiency – ❌ Also adrenal; leads to cortisol/sex steroid issues
1α-hydroxylase overexpression – ❌ Would cause hypercalcemia , not PTH elevation
Insulin has two chains (A and B) connected by disulfide bridges.
70 / 109
Category:
Endo – Biochemistry
Which statement is correct regarding insulin’s structure?
Insulin is a peptide hormone composed of two polypeptide chains :
A-chain (21 amino acids)
B-chain (30 amino acids)
These are linked together by two disulfide bonds , and an additional intra-chain disulfide bond is present within the A-chain. Since the two chains are different, this makes insulin a heterodimeric protein (i.e., a protein made of two different subunits).
It’s not a glycolipid, amino acid, homodimer, or steroid. Steroid hormones are derived from cholesterol and are lipophilic , unlike peptide hormones like insulin.
5) Answer Breakdown:
It is a heterodimeric protein – ✅ Correct; two different chains (A and B)
It is a glycolipid – ❌ Insulin is a protein, not lipid-based
It is an amino acid – ❌ It is made of amino acids but is not a single one
It is a homodimeric protein – ❌ Wrong; A and B chains are different
It is a steroid hormone – ❌ Peptide hormone, not steroid (not derived from cholesterol)
Think of “fight or flight”—these cells dump adrenaline and noradrenaline straight into the blood.
71 / 109
Category:
Endo – Histology
What are the hormone-secreting cells of the adrenal medulla known as?
The adrenal medulla is a modified sympathetic ganglion that secretes catecholamines (epinephrine and norepinephrine) directly into the bloodstream. The chromaffin cells are neuroendocrine cells that act like postganglionic sympathetic neurons, but instead of having axons, they release hormones into blood vessels.
Why the name “chromaffin”? Because these cells stain brown with chromium salts due to the oxidation of catecholamines—thus “chrom-affin” (chrome-loving).
They are derived from neural crest cells , like other parts of the sympathetic nervous system.
5) Answer Breakdown:
Chromaffin cells – ✅ Correct; secrete catecholamines in adrenal medulla
Melanocytes – ❌ Pigment-producing skin cells, also neural crest-derived but not endocrine
Pituicytes – ❌ Glial cells of the posterior pituitary, not adrenal
Corticocytes – ❌ Made-up or confused term; cortex cells are “cortical cells” but not used like this
Somatotropes – ❌ Anterior pituitary cells that secrete growth hormone (GH)
Glucagon is a peptide hormone , not a steroid or lipid—think pancreas, think alpha cells.
72 / 109
Category:
Endo – Physio
Which of the following statements is true regarding the synthesis of glucagon?
Glucagon is a peptide hormone made by the alpha cells of the islets of Langerhans in the pancreas. It’s first synthesized as a larger precursor called proglucagon , which is then cleaved to form active glucagon.
This is part of the typical pathway for peptide hormones:
Transcribed from DNA → mRNA
Translated into preprohormone (preproglucagon)
Processed in the ER/Golgi to proglucagon
Final cleavage in secretory vesicles → glucagon
This hormone raises blood glucose levels by promoting glycogen breakdown and gluconeogenesis, especially during fasting.
Answer Breakdown:
It is a steroid – ❌ Incorrect; steroids are lipid-based, glucagon is a peptide
It is synthesized as proglucagon in alpha cell – ✅ Correct; classic pathway for peptide hormones
It is a fatty acid – ❌ Nope, completely unrelated class
None of these – ❌ False, one is correct
It is synthesized from insulin – ❌ Wrong; insulin and glucagon are made by different pancreatic cells (beta vs alpha)
Think of where lipid-soluble hormones go—straight to the nucleus!
73 / 109
Category:
Endo – Physio
Steroid hormones bind to which of the following structures?
Steroid hormones (like cortisol, aldosterone, estrogen, progesterone, testosterone) are lipid-soluble , which means they diffuse through the cell membrane and bind to intracellular receptors —either in the cytoplasm or nucleus .
Once bound, the hormone-receptor complex enters the nucleus (if not already there) and binds to specific DNA sequences called Hormone Response Elements (HREs) . This interaction directly alters gene transcription , increasing or decreasing the expression of certain genes.
In contrast:
G-protein coupled receptors and enzyme-linked receptors are for peptide and amino acid-based hormones (like TSH, insulin, epinephrine).
Second messengers like cAMP or IP₃ are intermediate molecules , not binding sites.
5) Answer Breakdown:
Hormone response element in DNA – ✅ Correct; steroid hormones affect gene transcription via DNA binding
Enzyme linked receptors – ❌ Used by insulin and growth factors, not steroids
G-protein coupled receptors – ❌ Used by peptide hormones like glucagon
None of these – ❌ Incorrect, there is a correct answer
Second messengers – ❌ Used by non-steroid hormones; steroid hormones skip this
Think of the system that increases heart rate, dilates pupils, and redirects blood to muscles.
74 / 109
Category:
Endo – Physio
What part of the nervous system is activated in a fight or flight situation?
The sympathetic division of the autonomic nervous system is responsible for the body’s “fight or flight” response . When the brain perceives a threat, this system kicks in to prepare the body for rapid action. It does this by:
Increasing heart rate and blood pressure
Dilating bronchi and pupils
Shutting down non-essential functions like digestion
Releasing epinephrine and norepinephrine from the adrenal medulla
In contrast, the parasympathetic system is for “rest and digest.” The enteric nervous system controls gut function independently, while the somatic nervous system controls voluntary skeletal muscles.
5) Answer Breakdown:
Sympathetic part of autonomic nervous system – ✅ Correct; activates fight or flight
Parasympathetic part of autonomic nervous system – ❌ Rest and digest
Enteric nervous system – ❌ Gut regulation, not stress response
Special senses – ❌ Sensory input, not motor response
Somatic nervous system – ❌ Controls voluntary muscles, not autonomic response
Think of the system that increases heart rate, dilates pupils, and redirects blood to muscles.
75 / 109
Category:
Endo – Physio
What part of the nervous system is activated in a fight or flight situation?
The sympathetic division of the autonomic nervous system is responsible for the body’s “fight or flight” response . When the brain perceives a threat, this system kicks in to prepare the body for rapid action. It does this by:
Increasing heart rate and blood pressure
Dilating bronchi and pupils
Shutting down non-essential functions like digestion
Releasing epinephrine and norepinephrine from the adrenal medulla
In contrast, the parasympathetic system is for “rest and digest.” The enteric nervous system controls gut function independently, while the somatic nervous system controls voluntary skeletal muscles.
5) Answer Breakdown:
Sympathetic part of autonomic nervous system – ✅ Correct; activates fight or flight
Parasympathetic part of autonomic nervous system – ❌ Rest and digest
Enteric nervous system – ❌ Gut regulation, not stress response
Special senses – ❌ Sensory input, not motor response
Somatic nervous system – ❌ Controls voluntary muscles, not autonomic response
Think of the system that increases heart rate, dilates pupils, and redirects blood to muscles.
76 / 109
Category:
Endo – Physio
What part of the nervous system is activated in a fight or flight situation?
The sympathetic division of the autonomic nervous system is responsible for the body’s “fight or flight” response . When the brain perceives a threat, this system kicks in to prepare the body for rapid action. It does this by:
Increasing heart rate and blood pressure
Dilating bronchi and pupils
Shutting down non-essential functions like digestion
Releasing epinephrine and norepinephrine from the adrenal medulla
In contrast, the parasympathetic system is for “rest and digest.” The enteric nervous system controls gut function independently, while the somatic nervous system controls voluntary skeletal muscles.
5) Answer Breakdown:
Sympathetic part of autonomic nervous system – ✅ Correct; activates fight or flight
Parasympathetic part of autonomic nervous system – ❌ Rest and digest
Enteric nervous system – ❌ Gut regulation, not stress response
Special senses – ❌ Sensory input, not motor response
Somatic nervous system – ❌ Controls voluntary muscles, not autonomic response
Think of the system that increases heart rate, dilates pupils, and redirects blood to muscles.
77 / 109
Category:
Endo – Physio
What part of the nervous system is activated in a fight or flight situation?
The sympathetic division of the autonomic nervous system is responsible for the body’s “fight or flight” response . When the brain perceives a threat, this system kicks in to prepare the body for rapid action. It does this by:
Increasing heart rate and blood pressure
Dilating bronchi and pupils
Shutting down non-essential functions like digestion
Releasing epinephrine and norepinephrine from the adrenal medulla
In contrast, the parasympathetic system is for “rest and digest.” The enteric nervous system controls gut function independently, while the somatic nervous system controls voluntary skeletal muscles.
5) Answer Breakdown:
Sympathetic part of autonomic nervous system – ✅ Correct; activates fight or flight
Parasympathetic part of autonomic nervous system – ❌ Rest and digest
Enteric nervous system – ❌ Gut regulation, not stress response
Special senses – ❌ Sensory input, not motor response
Somatic nervous system – ❌ Controls voluntary muscles, not autonomic response
All steroid hormones—including cortisol, aldosterone, estrogen, and testosterone—begin their synthesis from a lipid molecule.
78 / 109
Category:
Endo – Biochemistry
Which of the following substances is the precursor for all steroid hormones?
Cholesterol is the universal precursor for all steroid hormones. It is taken up by steroidogenic cells (like those in the adrenal cortex or gonads) and converted into pregnenolone by the enzyme cholesterol desmolase (CYP11A1) in the mitochondria. Pregnenolone then serves as the branching point for the synthesis of various steroid hormones like glucocorticoids, mineralocorticoids, and sex hormones.
While pregnenolone and pregnanediol are important intermediates in the pathway, only cholesterol is the original source molecule. Amino acids like valine are unrelated to this pathway.
5) Answer Breakdown:
Valine – Amino acid; unrelated to steroid synthesis.
Cholesterol – ✅ Correct . It’s the base precursor for all steroid hormones.
Pregnanediol – A metabolite of progesterone, not a precursor.
Estrogen – A steroid hormone itself, not a precursor.
Pregnenolone – First product after cholesterol, but not the initial precursor.
This gland is known as the “master gland” because it controls several other hormone-secreting glands.
79 / 109
Category:
Endo – Physio
Which one of the following is an endocrine gland?
The pituitary gland is a pure endocrine gland , meaning it releases hormones directly into the bloodstream without using ducts. It has two major parts:
Anterior pituitary (adenohypophysis): Secretes hormones like ACTH, TSH, GH, LH, FSH, and prolactin .
Posterior pituitary (neurohypophysis): Stores and releases ADH and oxytocin produced by the hypothalamus.
It is under the regulatory control of the hypothalamus , and it influences growth, metabolism, reproduction , and more.
🧠 Answer Breakdown:
Pituitary ✅ → Correct. Classic endocrine gland; releases multiple essential hormones.
Spleen ❌ → Part of the immune system ; not endocrine.
Liver ❌ → Has endocrine-like functions (e.g., IGF-1 secretion), but not classified as a pure endocrine gland.
Skin ❌ → Can produce vitamin D precursors , but it’s not an endocrine gland.
Tongue ❌ → Involved in taste and mechanical digestion ; no hormonal function.
This hormone inhibits insulin and glucagon secretion, and comes in both 14 and 28 amino acid forms.
80 / 109
Category:
Endo – Biochemistry
Which polypeptide is made up of 14 amino acids and is also found in the pancreas?
Somatostatin is a regulatory hormone found in both the hypothalamus and the pancreas . It exists in two main forms:
In the pancreas , somatostatin is secreted by the delta (δ) cells of the islets of Langerhans. It acts locally to inhibit:
Insulin secretion from beta cells
Glucagon secretion from alpha cells
Pancreatic enzyme and bicarbonate secretion
It’s a paracrine hormone , meaning it influences nearby cells.
🧠 Answer Breakdown:
Somatostatin ✅ → Correct. 14-amino-acid form in the pancreas; inhibits other pancreatic hormones.
Cortisol ❌ → A steroid hormone, not a polypeptide; secreted by adrenal cortex.
Insulin ❌ → A 51-amino-acid polypeptide; not 14.
Glucagon ❌ → A 29-amino-acid polypeptide.
Pancreatic polypeptide ❌ → Composed of 36 amino acids , secreted by PP cells.
Think: before anything else, the thyroid must collect the raw material — iodide — from the blood.
81 / 109
Category:
Endo – Biochemistry
What is the first step in thyroid hormone synthesis?
The first step in thyroid hormone synthesis is iodide trapping , also called iodide uptake . This process involves actively transporting iodide ions (I⁻) from the bloodstream into the follicular cells of the thyroid gland using a sodium-iodide symporter (NIS) .
Only after iodide is trapped inside the follicular cell can it be:
Oxidized to iodine (I₂)
Organified (attached to tyrosine residues on thyroglobulin)
Coupled to form T₃ and T₄
Stored , then later released
🧠 Answer Breakdown:
Iodide trapping ✅ → Correct. This is the essential first step.
Organification ❌ → Occurs after iodide is oxidized to iodine.
Splitting ❌ → This refers to the breakdown of T₄ or T₃, not part of synthesis.
Conjugation ❌ → Refers to coupling of iodinated tyrosines (MIT + DIT) — a later step.
Deiodination ❌ → Happens in peripheral tissues for recycling iodine or converting T₄ to T₃.
Think about the adrenal cortex vs the medulla — they come from different germ layers.
82 / 109
Category:
Endo – Embryology
What layer(s) is the adrenal gland derived from?
The adrenal gland develops from two embryological layers :
The adrenal cortex arises from the mesoderm (specifically intermediate mesoderm).
The adrenal medulla comes from ectoderm , more specifically from neural crest cells , which are ectodermal in origin.
This dual origin is important because the cortex secretes steroid hormones (like cortisol and aldosterone), while the medulla secretes catecholamines (like adrenaline), which are under neural control.
🧠 Answer Breakdown:
Only mesoderm ❌ → Incorrect. Only gives rise to cortex, not medulla.
Only endoderm ❌ → Incorrect. Adrenal gland has no endodermal contribution.
Ectoderm and endoderm ❌ → Incorrect. Endoderm is not involved in adrenal development.
Ectoderm and mesoderm ✅ → Correct. Medulla = ectoderm (neural crest), Cortex = mesoderm.
Only ectoderm ❌ → Incorrect. Covers medulla only, misses cortex.
Think about fetal development — when do organs like the thyroid form?
83 / 109
Category:
Endo – Embryology
Which statement is wrong regarding the thyroid gland?
The thyroid gland develops early during fetal life , not after birth . In fact, it’s the first endocrine gland to develop , around the 4th week of gestation . It begins as an endodermal diverticulum from the floor of the primitive pharynx (at the site of the foramen cecum on the tongue), and descends to its final position in the neck.
All the other options describe correct features of the thyroid gland:
It is supplied by the inferior thyroid artery (a branch of the thyrocervical trunk).
It lies approximately at C5–T1 vertebral level .
It originates from the thyroid diverticulum .
The thyroglossal duct is a temporary embryonic structure connecting the thyroid to the tongue during development and can sometimes persist abnormally.
🧠 Answer Breakdown:
It is supplied by the inferior thyroid artery ✅ → True. Arises from thyrocervical trunk.
It is at the vertebral level of C5-T1 ✅ → True. Anatomically accurate.
It develops from a diverticulum in the tongue ✅ → True. From the foramen cecum region.
It may form a thyroglossal duct ✅ → True. A developmental remnant.
It develops after birth ❌ → False. The thyroid forms during embryogenesis , not after birth.
Think of the brain’s “master controller” that links the nervous system to the endocrine system.
84 / 109
Category:
Endo – Physio
What structure controls the pituitary gland?
The hypothalamus is the primary regulator of the pituitary gland , often called the “master of the master gland.” It maintains homeostasis by integrating neural signals and releasing hormones that either stimulate or inhibit the anterior pituitary.
It releases releasing hormones (like TRH, CRH, GnRH, GHRH) and inhibitory hormones (like somatostatin and dopamine) into the hypophyseal portal system , directly controlling anterior pituitary activity.
It also directly sends neurosecretory axons into the posterior pituitary , where hormones like ADH and oxytocin are stored and released.
Thus, the hypothalamus is the key brain structure that controls the pituitary gland and, through it, the entire endocrine system .
🧠 Answer Breakdown:
Cerebellum ❌ → Coordinates movement and balance. Not involved in endocrine control.
Thalamus ❌ → Relay station for sensory information; not the pituitary controller.
Pancreas ❌ → Endocrine organ, but not related to pituitary regulation.
Hypothalamus ✅ → Correct. Master regulator of the pituitary gland.
Adrenal gland ❌ → Regulated by the pituitary via ACTH, not the other way around.
These cells support the axons of neurosecretory neurons from the hypothalamus, not hormone-secreting cells themselves.
85 / 109
Category:
Endo – Histology
What are the glial cells of the posterior pituitary also known as?
The posterior pituitary (neurohypophysis) is actually an extension of the hypothalamus . It stores and releases oxytocin and vasopressin (ADH), which are synthesized in the hypothalamic nuclei (supraoptic and paraventricular).
The glial cells of the posterior pituitary are called pituicytes . They function similarly to astrocytes —supporting and insulating the axons of the neurosecretory neurons. Pituicytes do not produce hormones ; they assist in the storage and release of hypothalamic hormones into the bloodstream.
🧠 Answer Breakdown:
Somatotropes ❌ → Anterior pituitary cells that secrete growth hormone (GH) .
Melanocytes ❌ → Pigment-producing skin cells, unrelated to the pituitary.
Mammotropes ❌ → Anterior pituitary cells that secrete prolactin .
Gonadotropes ❌ → Anterior pituitary cells that secrete LH and FSH .
Pituicytes ✅ → Correct answer. These are the support glial cells of the posterior pituitary .
ACTH is a peptide hormone . Most peptide hormones act through G-protein coupled receptors (GPCRs) and use second messengers like cAMP.
86 / 109
Category:
Endo – Physio
What is the mechanism of action for adrenocorticotropic hormone (ACTH)?
ACTH (adrenocorticotropic hormone), secreted by the anterior pituitary, binds to melanocortin 2 receptors (MC2R) on adrenal cortex cells . These receptors are Gs protein-coupled receptors , which means:
ACTH binds to MC2R.
This activates the Gs alpha subunit .
Gs stimulates adenylyl cyclase , which converts ATP to cAMP .
cAMP activates protein kinase A (PKA) .
PKA phosphorylates proteins that increase steroidogenesis , especially cortisol synthesis .
This is the classic cAMP pathway , used by many anterior pituitary hormones.
🧠 Answer Breakdown:
Intracellular receptor pathway ❌ → Used by steroid hormones (e.g. cortisol, estrogen), not ACTH.
cAMP as a second messenger ✅ → Correct for ACTH.
Tyrosine kinase linked ❌ → Used by insulin and growth factors (e.g. IGF-1), not ACTH.
Phospholipase linked pathway ❌ → Refers more to PLC/IP3/DAG pathways, used by hormones like TRH .
IP3 linked pathway ❌ → Also incorrect; ACTH doesn’t use IP3/Ca²⁺.
📌 Clinical Note: Mutations in the MC2R or Gs protein can lead to ACTH resistance , resulting in familial glucocorticoid deficiency .
Most peptide and protein hormones use cAMP as a second messenger. But steroid hormones act via intracellular receptors , bypassing the second messenger system.
87 / 109
Category:
Endo – Physio
Which of the following hormones does not use cAMP as a second messenger?
Hormones exert their effects via different signaling pathways :
Peptide hormones (like TSH, ACTH, FSH, glucagon) are water-soluble and act via membrane receptors , often triggering second messengers like cAMP through G-protein coupled receptors (GPCRs) .
Steroid hormones , such as estrogen , are lipid-soluble and diffuse directly into cells , binding to intracellular nuclear receptors . They do not use second messengers like cAMP.
Instead, estrogen acts as a transcription factor , altering gene expression directly by binding to estrogen response elements (EREs) in DNA.
🧠 Answer Breakdown:
TSH ❌ → Uses cAMP via Gs protein.
Glucagon ❌ → Strong cAMP activator in liver for glycogenolysis.
Estrogen ✅ → Steroid hormone , no cAMP . Acts via intracellular receptors .
ACTH ❌ → Binds to MC2 receptor → Gs → cAMP in adrenal cortex.
FSH ❌ → Binds to Gs-coupled receptor → cAMP ↑ in gonads.
📌 Clinical Tip: Drugs or toxins affecting adenylyl cyclase or G proteins can disrupt hormone action for all the ones using cAMP — but not for steroid hormones like estrogen.
Only one endocrine gland stores its hormone extracellularly in a structure called a follicle , filled with a colloid substance rich in thyroglobulin .
88 / 109
Category:
Endo – Histology
Which of the following glands has follicles with colloid?
The thyroid gland is unique among endocrine glands in its follicular architecture . It contains spherical follicles , each lined by a single layer of epithelial (follicular) cells , and filled with a gel-like colloid in the center.
This colloid is primarily composed of thyroglobulin , a precursor protein for T3 (triiodothyronine) and T4 (thyroxine) .
Unlike most endocrine glands that store hormones intracellularly , the thyroid stores large quantities of inactive thyroid hormone precursor (thyroglobulin) extracellularly in the colloid.
🧠 Breakdown of the Options:
Sebaceous glands ❌
Parathyroid ❌
Pancreas ❌
Adrenal gland ❌
Thyroid ✅ (Correct)
📌 Extra Clinical Tip: In conditions like Graves disease , these follicles become hyperactive and colloid is scanty . In hypothyroidism , colloid may be abundant due to reduced hormone secretion.
When a gland is working less , its lining cells tend to appear flatter and less metabolically active. When it’s working more , would you expect the lining cells to be tall and busy — or thin and passive?
89 / 109
Category:
Endo – Physio
Regarding the squamous epithelium of the thyroid, which of the following statements is false?
The thyroid gland is made up of spherical follicles lined by epithelial cells and filled with colloid (stored thyroglobulin). The type of epithelium lining the follicles reflects the functional state of the gland :
🔄 Thyroid Activity and Epithelial Morphology:
Thyroid State
Follicular Epithelium Type
Colloid Appearance
Normal
Simple cuboidal
Moderate amount
Hyperactive
Simple columnar
Scanty, often scalloped
Hypoactive
Simple squamous or flattened
Abundant, dense colloid
In some long-standing hypoactive states or conditions like Hashimoto thyroiditis , metaplastic changes may lead to areas lined by stratified squamous epithelium — but this is not the norm , and typically indicates chronic damage or degeneration , not heightened function.
So, saying that squamous epithelium is found in a hyperactive thyroid is false — in fact, hyperactivity causes the opposite: the cells become taller and more metabolically active.
🔍 Answer Breakdown:
It shows in a hyperactive thyroid ❌ (False)
It can be stratified squamous epithelium ✅ (True)
It is accompanied by zero to low colloid ✅ (True)
It transitions to columnar in hyperactive thyroid ✅ (True)
It shows in a hypoactive thyroid ✅ (True)
Think about the hormone your body leans on in times of stress, trauma, or fasting — not just emotionally, but physiologically. Without it, even mild illness or injury can tip the body into collapse due to impaired blood sugar regulation and vascular tone.
90 / 109
Category:
Endo – Physio
A deficiency of which of the following hormones can cause a life-threatening condition?
Cortisol is a glucocorticoid hormone secreted by the zona fasciculata of the adrenal cortex . It plays a vital role in maintaining homeostasis , especially during stressful conditions such as illness, trauma, or fasting.
🌡️ Functions of Cortisol:
Increases blood glucose via gluconeogenesis
Enhances vascular responsiveness to catecholamines (helps maintain blood pressure)
Suppresses inflammation and immune response
Helps in fat, protein, and carbohydrate metabolism
⚠️ Why Cortisol Deficiency is Life-Threatening:
In conditions like Addison’s disease (primary adrenal insufficiency), cortisol production is severely impaired. This can lead to adrenal crisis , a medical emergency characterized by:
Cortisol is essential for:
Maintaining blood pressure
Ensuring adequate glucose availability (especially during fasting)
Counteracting stress responses
Even minor infections or surgeries can be fatal in someone with cortisol deficiency if not managed properly with steroid replacement.
🔍 Answer Breakdown:
Cortisol ✅
Aldosterone ❌
Also critical, especially for sodium/potassium balance. However, acute cortisol deficiency can more directly impair vascular tone and glucose balance, leading to crisis.
Insulin ❌
Thyroxine ❌
Testosterone ❌
Think about the enzyme that acts as a signal amplifier in many hormonal pathways — it doesn’t just relay the message; it transforms a high-energy molecule into a powerful second messenger that drives the cell’s response.
91 / 109
Category:
Endo – Physio
What is the function of adenylyl cyclase?
Adenylyl cyclase (also called adenylate cyclase ) is a key enzyme in many signal transduction pathways , especially those involving G-protein coupled receptors (GPCRs) .
🔁 How It Works:
A ligand (like a hormone) binds to a GPCR on the cell surface.
This activates a G protein (specifically, Gs for stimulatory or Gi for inhibitory).
The Gs protein activates adenylyl cyclase .
Adenylyl cyclase then catalyzes the conversion of:
🎯 Role of cAMP:
cAMP acts as a second messenger that:
Activates protein kinase A (PKA)
Leads to phosphorylation of target proteins
Amplifies the signal initiated by the original ligand
This pathway is used by hormones such as:
Thus, adenylyl cyclase is the enzyme that links receptor activation to cellular response via cAMP production .
🔍 Answer Breakdown:
Conversion of ATP to cAMP ✅
Activation of G protein ❌
Conversion of ATP to cGMP ❌
Conversion of ATP to AMP ❌
Conversion of cAMP to ATP ❌
Think about the stigma around seeking help — especially when it’s assumed that doing so must mean something is seriously wrong . But is that really the only reason someone might benefit from a listening ear and structured support?
92 / 109
Category:
Endo – Community Medicine/Behavioral Sciences
What is the most common misconception about counseling?
Counseling is a structured, supportive interaction that helps individuals cope with personal, social, emotional, or psychological challenges. It’s a preventive , interventive , and supportive tool — not solely a treatment for mental illness.
🧠 Common Misconception:
“Counseling is only for people with psychiatric disorders.”
This belief is widespread and incorrect . It leads to:
Stigma about seeking help
Delays in intervention for people facing life stressors
Undermining the role of preventive mental health
In truth, counseling is:
For stress management , career guidance , relationship issues , grief , and personal growth
Used in schools , workplaces , and healthcare settings
A tool for building resilience , improving communication , and enhancing decision-making
Just like you don’t need to have heart disease to visit a cardiologist for a check-up, you don’t need to have a psychiatric disorder to benefit from counseling.
🔍 Answer Breakdown:
Counseling is only for psychiatric disorders ✅
Counseling is done for mental problems ❌
Counseling is not necessary for good mental health ❌
Counseling not only benefits you but also your loved ones ❌
None of these ❌
Think about the cells responsible for regulating blood sugar levels after a meal . Which ones release a hormone that tells tissues to absorb glucose and promotes storage rather than release?
93 / 109
Category:
Endo – Physio
What type of cells secrete insulin?
The pancreas has both exocrine and endocrine components. The endocrine portion consists of the Islets of Langerhans , which contain specialized hormone-secreting cells . Each type plays a specific role in glucose homeostasis and digestion:
🔬 Cell Types in Islets of Langerhans:
Cell Type
Hormone Secreted
Function
Alpha (α)
Glucagon
Raises blood glucose
Beta (β)
Insulin
Lowers blood glucose by promoting glucose uptake and storage
Delta (δ)
Somatostatin
Inhibits insulin, glucagon, and growth hormone
PP (F)
Pancreatic polypeptide
Regulates pancreatic secretions and GI motility
I cells
CCK (Cholecystokinin)
Found in small intestine, not pancreas
🧪 Insulin:
A loss of beta-cell function results in Type 1 Diabetes Mellitus (autoimmune destruction), and impaired function contributes to Type 2 Diabetes Mellitus .
🔍 Answer Breakdown:
Beta cells ✅
Alpha cells ❌
Delta cells ❌
PP cells (F cells) ❌
I cells ❌
Think about the body’s minimal daily requirement of a trace element needed to support both T3 and T4 synthesis — not an excessive supplement, just enough to prevent goiter and hypothyroidism in an average adult.
94 / 109
Category:
Endo – Physio
What quantity of iodine is required per day for the normal production of thyroid hormones?
Iodine is a crucial component of thyroid hormones :
The body cannot synthesize iodine , so it must be obtained from the diet (e.g., iodized salt, seafood, dairy). Once absorbed, iodine is concentrated by the thyroid gland , where it’s used to iodinate tyrosine residues on thyroglobulin — a key step in forming T3 and T4.
🧂 Recommended Daily Allowance (RDA):
According to the World Health Organization (WHO) and Endocrine Society :
Adults need about 150 µg (micrograms) per day.
Pregnant/lactating women require more: around 220–290 µg/day
Children require less, depending on age.
Insufficient iodine intake leads to:
That’s why iodized salt programs are a major public health intervention globally.
🔍 Answer Breakdown:
150 microgram ✅
50 microgram ❌
250 microgram ❌
350 microgram ❌
500 microgram ❌
In a condition where the thyroid is overactive , producing excess hormone without the pituitary’s instruction , what happens to the hormone that normally tells the thyroid what to do?
95 / 109
Category:
Endo – Pathology
What is the test for Graves disease?
Graves disease is the most common cause of hyperthyroidism . It’s an autoimmune disorder in which the body produces TSH receptor antibodies (TRAb) . These antibodies mimic TSH , stimulating the thyroid gland to secrete excess T3 and T4 , even though TSH from the pituitary is not needed .
🧪 What happens to TSH levels?
This makes TSH concentration the best initial test when evaluating a patient with suspected thyroid dysfunction — both hypo- and hyperthyroidism .
If TSH is low , and symptoms point toward hyperthyroidism, the next steps may include:
🔍 Answer Breakdown:
To diagnose a condition characterized by excess of a hormone, would it make sense to check just one momentary value, or would a test that captures hormone production over a greater amount of time give a more reliable picture?
96 / 109
Category:
Endo – Pathology
What is the best diagnostic test for Cushing syndrome?
Cushing syndrome is a clinical condition resulting from chronic exposure to excessive cortisol . It can be:
🎯 Diagnosing Cushing Syndrome: Why a 24-hour urine cortisol test?
Because cortisol levels fluctuate throughout the day — peaking in the early morning and dropping by night — a single blood or saliva measurement may miss the diagnosis. That’s why a test that measures total cortisol output over time is more reliable for initial screening .
The 24-hour urinary free cortisol test reflects the unbound, biologically active cortisol excreted in the urine. In Cushing syndrome , levels will be consistently elevated across the collection period.
This test is:
Non-invasive
Not affected by diurnal variation
Recommended as a first-line screening test by clinical guidelines (e.g., Endocrine Society)
🩺 What comes next?
Once Cushing syndrome is confirmed, further tests (e.g., ACTH levels, dexamethasone suppression tests, CRH stimulation) are used to determine the cause — whether it’s:
🔍 Answer Breakdown:
24-hour urine collection for free cortisol ✅
Adrenocorticotropic hormone (ACTH) levels test ❌
Serum cortisol levels test ❌
Cortisol is subject to diurnal variation . A one-time blood test may give a false-negative or false-positive .
Corticotropin-releasing hormone (CRH) stimulation test ❌
A secondary test , used to distinguish between pituitary and ectopic ACTH production . Not used initially.
High-dose dexamethasone suppression test ❌
Also a second-line test used to determine cause , not to confirm presence of Cushing syndrome.
Imagine a gland working overtime — making and secreting large amounts of hormone. Would its lining cells appear flattened and passive, or tall and actively engaged in producing and transporting substances?
97 / 109
Category:
Endo – Histology
What type of epithelial cells will be seen in a hyperactive thyroid?
The thyroid gland is made of follicles , which are spherical structures filled with colloid (a protein-rich substance primarily containing thyroglobulin).
These follicles are lined by epithelial cells whose shape reflects the gland’s activity level .
🔁 Activity-Dependent Changes:
Thyroid Activity
Epithelial Cell Type
Hypoactive
Simple squamous
Normal
Simple cuboidal
Hyperactive
Simple columnar
In a hyperactive thyroid (e.g., Graves’ disease or TSH overstimulation ):
Follicular cells increase in height to a simple columnar shape.
The colloid is reabsorbed , giving follicles a scalloped appearance (a hallmark in Graves’ disease).
These taller cells indicate active synthesis and secretion of thyroid hormones (T3, T4).
This change is adaptive — taller cells allow more organelles like mitochondria, RER, and Golgi to function, supporting increased protein synthesis (thyroglobulin) and hormone production.
🔍 Answer Breakdown:
Imagine a region where prenatal nutrition is not optimal, especially in trace minerals. Now, think about the one specific element required for synthesizing the very hormone that governs growth, brain development, and metabolism in utero. What happens if the mother’s body lacks this key component?
98 / 109
Category:
Endo – Physio
What is the most common cause of congenital hypothyroidism worldwide?
Congenital hypothyroidism is a condition present at birth where the thyroid gland fails to produce sufficient thyroid hormones. These hormones — T3 and T4 — are critical for brain development , especially in the first few years of life. If untreated, congenital hypothyroidism can lead to intellectual disability and growth retardation , historically known as cretinism .
Causes can be broadly divided into:
Primary thyroid gland problems (e.g., agenesis, ectopy, dyshormonogenesis)
Iodine deficiency
Globally, the most common cause is iodine deficiency — particularly in areas where iodized salt is not used or where the soil is deficient in iodine (e.g., mountainous regions of South Asia, Central Africa).
Iodine is an essential component of thyroid hormones :
T3 has 3 iodine atoms
T4 has 4 iodine atoms
When the mother’s iodine intake is insufficient during pregnancy, the fetus cannot synthesize enough thyroid hormone , even if the gland is structurally normal. This is why universal salt iodization is one of the biggest public health interventions worldwide — to prevent neurodevelopmental disorders in children.
In developed countries, where iodine intake is adequate, thyroid dysgenesis (failure of thyroid gland development) is the most common cause — but worldwide , iodine deficiency leads the list .
🔍 Answer Breakdown:
Iodine deficiency ✅
Dyshormonogenetic goiter ❌
Lack of iodine intake in childhood ❌
Hereditary ❌
Failure of thyroid gland to develop ❌
Think about how the body manages to circulate hormones that are lipophilic and not easily soluble in water-based plasma. What kind of helper molecule would it need to carry them safely through the bloodstream to their target tissues?
99 / 109
Category:
Endo – Physio
What substance transports thyroid hormones in blood?
Thyroid hormones — T3 (triiodothyronine) and T4 (thyroxine) — are lipid-soluble molecules. This makes them poorly soluble in plasma, which is largely water-based. To circulate efficiently, they need to be bound to plasma proteins that serve as carriers.
The primary transport protein for thyroid hormones is Thyroxine-Binding Globulin (TBG) . It has:
Other proteins that contribute to transport (with lower affinity but higher capacity) include:
However, TBG is the main and most clinically significant carrier. Importantly, only free (unbound) T3 and T4 are biologically active — but bound hormone serves as a reservoir , buffering rapid fluctuations in levels.
🔍 Answer Breakdown:
Consider the body’s strategy for regulating active thyroid hormone levels. Instead of synthesizing everything from scratch, it often modifies a more abundant, less active molecule by removing a specific atom — a process that allows rapid local control of activity without requiring new hormone production.
100 / 109
Category:
Endo – Physio
What process contributes to the synthesis of most T3?
The thyroid gland secretes mostly T4 (about 90%) and a small amount of T3 directly (about 10%). However, T3 is the biologically active form of thyroid hormone — it binds with much higher affinity to nuclear receptors and has a more potent physiological effect than T4.
So why secrete mostly T4? Because T4 serves as a prohormone , a reservoir that can be converted to T3 as needed , especially in peripheral tissues like the liver, kidneys, and muscles.
How is this conversion done?
By deiodination — the removal of one iodine atom from T4 (which has 4 iodine atoms), converting it to T3 (which has 3 iodine atoms). This is catalyzed by iodothyronine deiodinases , of which there are three types:
This system allows fine-tuned control of thyroid hormone activity at the tissue level , depending on local needs.
Deiodination of T4 ✅
Deamination of T4 ❌
Reduction of T4 ❌
Oxidation of T4 ❌
Reiodination of T4 ❌
Consider a scenario where hypertension arises not from typical vascular resistance or fluid overload, but from sudden surges of a powerful endogenous substance that stimulates both alpha and beta receptors. In such a case, would standard antihypertensives suffice — or would you need something more targeted?
101 / 109
Category:
Endo – Pharmacology
Phentolamine is used to reduce hypertension in which of the following conditions?
Phentolamine is a non-selective alpha-adrenergic antagonist — it blocks both α1 and α2 receptors . This causes:
Vasodilation (via α1 blockade), reducing peripheral vascular resistance and thus lowering blood pressure.
Increased norepinephrine release (via α2 blockade), which can lead to reflex tachycardia.
Because of these effects, phentolamine is not a routine antihypertensive . Instead, it’s reserved for special hypertensive crises , particularly those involving excess catecholamines .
🧬 Enter: Pheochromocytoma
Pheochromocytoma is a catecholamine-secreting tumor of the adrenal medulla (producing epinephrine, norepinephrine, and sometimes dopamine ). This causes:
Paroxysmal or sustained hypertension
Palpitations, sweating, headache
Treating these patients with regular beta-blockers or standard antihypertensives can worsen the crisis if alpha-blockade isn’t done first. Why?
Because alpha receptors cause vasoconstriction . If you block only beta receptors (e.g., with propranolol), you’ll leave unopposed alpha activity — leading to severe vasoconstriction and hypertensive crisis .
Hence, phentolamine (or phenoxybenzamine) is used to:
✅ Correct Answer: Pheochromocytoma
❌ Why the Other Options Are Incorrect:
Hypercholesterolemia:
Diabetes mellitus:
May lead to hypertension through vascular and renal changes, but treatment is with ACE inhibitors, ARBs, or CCBs — not alpha-blockers like phentolamine.
Metabolic syndrome:
Associated with insulin resistance, obesity, and low-grade chronic hypertension. Again, not catecholamine-driven → phentolamine isn’t used.
None of these:
This hormone works quickly and precisely by binding to receptors and triggering rapid effects on metabolism. Its size reflects a balance — large enough to have receptor specificity, but small enough for quick synthesis and breakdown. Think about how many amino acids are needed for that level of function.
102 / 109
Category:
Endo – Physio
What is the number of amino acids found in insulin?
Insulin is a peptide hormone produced by the beta cells of the pancreatic islets of Langerhans . It is essential for:
Promoting glucose uptake into cells
Facilitating glycogen synthesis
Inhibiting gluconeogenesis and lipolysis
Its role in glucose homeostasis is vital, and disturbances in its production or action lead to diabetes mellitus .
🧬 Structure of Insulin:
Insulin has a total of 51 amino acids , organized into two chains :
A chain : 21 amino acids
B chain : 30 amino acids
These chains are connected by two disulfide bridges , and there is also one intra-chain disulfide bond within the A chain.
Insulin is originally synthesized as preproinsulin (single chain), which is cleaved in the Golgi apparatus into proinsulin , and finally processed into mature insulin + C-peptide.
❌ Why the Other Options Are Incorrect:
1. 10 amino acids
Much too short for insulin.
Only small signaling molecules like GnRH (gonadotropin-releasing hormone) have around 10 amino acids.
❌ Incorrect — insulin is much more complex.
2. 69 amino acids
This number doesn’t correspond to insulin or its precursors.
May be confused with other polypeptides or incorrectly recalled.
❌ Incorrect — too many amino acids for insulin’s known structure.
3. 191 amino acids
This is the number of amino acids in growth hormone , not insulin.
GH is a large, single-chain polypeptide hormone.
❌ Incorrect — GH, not insulin.
4. 89 amino acids
Still too large for insulin.
Might seem plausible if confusing insulin with proinsulin , but even proinsulin has only 86 amino acids , including the C-peptide.
❌ Incorrect — not the mature form of insulin.
Consider the functional complexity of a hormone responsible for widespread growth effects — from bones to muscles to metabolism. Would such a molecule likely be small and simple, or larger and structurally complex?
103 / 109
Category:
Endo – Physio
What is the number of amino acids found in the structure of growth hormone?
Growth Hormone (GH), also known as somatotropin , is a peptide hormone secreted by the anterior pituitary gland .
It plays a central role in stimulating linear growth , muscle mass increase , lipolysis , glucose metabolism , and organ development .
🧬 Structure of GH:
Human growth hormone is composed of a single-chain polypeptide .
It consists of 191 amino acids in its full-length, biologically active form.
This linear chain folds into a specific three-dimensional structure , stabilized by disulfide bridges , to bind effectively to the GH receptor.
This 191-amino-acid structure is highly conserved and essential for its function in stimulating IGF-1 (insulin-like growth factor 1) release, among other effects.
So, the correct and specific number of amino acids in GH is 191 .
❌ Why the Other Options Are Incorrect:
1. 69 amino acids
This might be confused with other smaller peptide hormones (e.g., some gastrointestinal peptides), but not GH .
❌ Incorrect — too short for the size and function of GH.
2. 10 amino acids
Extremely short — typical of tiny peptides like gonadotropin-releasing hormone (GnRH) .
GH is a much larger polypeptide .
❌ Incorrect — GH is not a decapeptide.
3. 112 amino acids
Again, not associated with GH, though some other proteins or cytokines fall in this range.
❌ Incorrect — GH is significantly larger.
4. 89 amino acids
Might confuse with parathyroid hormone (PTH) , which has 84 amino acids.
Still much shorter than the 191 amino acids in GH.
❌ Incorrect — not sufficient to form the structure and receptor interaction domains of GH.
Sometimes too much of a good thing can shut down the very system it’s supposed to support. Ask yourself: how might the body protect itself from the risk of overproduction when essential building blocks suddenly become overwhelmingly abundant?
104 / 109
Category:
Endo – Physio
Extremely high iodine levels lead to which of the following?
Iodine is an essential element required for the synthesis of thyroid hormones (T3 and T4). The thyroid gland actively transports iodine from the blood via the sodium-iodide symporter and incorporates it into hormone production.
However, excessive iodine intake —especially in sudden, large amounts—can paradoxically lead to decreased thyroid hormone production , a phenomenon known as the Wolff-Chaikoff effect .
🔬 Wolff-Chaikoff Effect (Key Concept)
When plasma iodine levels rise above a critical threshold , the thyroid gland temporarily shuts down hormone synthesis to protect against overproduction.
This autoregulatory response involves inhibition of iodide organification (the process by which iodine is added to thyroglobulin).
If this suppression persists, it can lead to hypothyroidism , especially in susceptible individuals (e.g., those with underlying autoimmune thyroid disease).
So, extremely high iodine levels → suppression of thyroid hormone synthesis → hypothyroidism .
❌ Why the Other Options Are Incorrect:
1. Exophthalmos
This is a classic feature of Graves’ disease , an autoimmune cause of hyperthyroidism .
While it is related to thyroid disease, it is not directly caused by iodine excess .
❌ Incorrect — not a result of high iodine levels.
2. Cushing syndrome
Caused by prolonged elevated cortisol levels , either from endogenous overproduction (e.g., adrenal tumor, pituitary adenoma) or exogenous steroids.
Has nothing to do with iodine or the thyroid gland .
❌ Incorrect — unrelated endocrine disorder.
3. Hyperthyroidism
Logical sounding, right? After all, iodine is needed to make thyroid hormones.
However, in the short term , very high iodine can actually inhibit thyroid function due to the Wolff-Chaikoff effect .
In some rare cases (like Jod-Basedow phenomenon), excess iodine can cause hyperthyroidism—but only in certain populations with nodular goiters or latent Graves’ disease .
In the general population, the more typical response to excess iodine is hypothyroidism .
❌ Incorrect in most physiological contexts.
4. Conn syndrome
Caused by aldosterone-producing adrenal tumors , leading to hyperaldosteronism .
Results in hypertension, hypokalemia, and metabolic alkalosis .
It is a disorder of the adrenal cortex , not related to iodine or thyroid function.
❌ Incorrect — different endocrine axis.
Think about the branching patterns of arteries in the lower neck. One structure in particular acts as a short trunk that gives rise to multiple branches directed toward the thyroid, scapula, and neck muscles. Visualizing the anterior aspect of the subclavian artery might help.
105 / 109
Category:
Endo – Anatomy
The inferior thyroid artery is a branch of which of the following structures?
The thyroid gland receives a rich blood supply from multiple arteries. Two main arteries supply the thyroid gland:
Superior thyroid artery
Inferior thyroid artery
Each of these arises from different parent vessels and at different levels in the neck.
🔎 Correct Option: Thyro-cervical trunk
The thyro-cervical trunk is a short arterial branch that arises from the subclavian artery , specifically from its first part .
It gives rise to three or four branches, one of which is the inferior thyroid artery .
The inferior thyroid artery ascends to supply the posterior and inferior parts of the thyroid gland , and also gives off branches to the parathyroid glands and surrounding musculature.
Thus, the inferior thyroid artery is a direct branch of the thyro-cervical trunk , which makes this the correct answer.
❌ Let’s Review the Incorrect Options:
1. Internal carotid artery
This artery does not give off any branches in the neck .
It ascends to enter the skull and supplies the brain, eyes, and forehead.
It plays no role in thyroid gland blood supply .
❌ Incorrect because it never branches before reaching the skull.
2. Internal thoracic artery
This artery is also a branch of the subclavian artery , like the thyro-cervical trunk.
However, it descends into the thorax , running along the inner side of the anterior chest wall.
It gives rise to anterior intercostal arteries , pericardiacophrenic arteries , and contributes to mammary gland and diaphragm supply —not the thyroid .
❌ Incorrect because it’s not anatomically related to thyroid vasculature.
3. External carotid artery
This artery does give off the superior thyroid artery , which supplies the upper pole of the thyroid gland.
However, the inferior thyroid artery is not one of its branches.
This is a commonly confused option , but it’s important to note the external carotid system stays more superior .
❌ Incorrect , despite being close in location and partially involved in thyroid supply.
4. Superior thyroid artery
This is a branch of the external carotid artery , and it supplies the upper part of the thyroid .
The inferior thyroid artery is a separate vessel and arises from a completely different source.
Students sometimes think it’s a branch because the names sound hierarchical (superior/inferior), but they come from distinct arteries .
❌ Incorrect because it’s a peer , not a parent artery to the inferior thyroid artery.
When the body faces a stressful situation, multiple systems are activated to mobilize energy rapidly. Which hormone works with cortisol in these conditions, not against it?
106 / 109
Category:
Endo – Physio
Which of the following hormones has effects most similar to cortisol?
Cortisol , a glucocorticoid hormone produced by the adrenal cortex, plays a major role in the body’s response to stress , particularly by increasing blood glucose levels , suppressing inflammation , and mobilizing energy stores .
When comparing hormones for similar effects , we look at physiological outcomes , such as:
Elevation of blood glucose
Promotion of lipolysis
Enhanced cardiovascular function
Stress response coordination
✅ Why “Epinephrine” is correct:
Epinephrine , like cortisol, is released in response to stress (fight or flight).
It is a catecholamine secreted by the adrenal medulla .
Shared effects with cortisol include:
Increased blood glucose (via glycogenolysis and gluconeogenesis)
Lipolysis to mobilize energy stores
Enhanced cardiovascular output
Although they act through different mechanisms (epinephrine via adrenergic receptors , cortisol via intracellular steroid receptors ), their net effects on metabolism and energy mobilization are complementary and overlapping .
❌ Why the other options are incorrect:
Insulin : Has opposite effects to cortisol. It promotes glucose uptake , storage , and anabolism , whereas cortisol promotes glucose release and catabolism .
Prolactin : Involved in lactation , not in energy metabolism or stress response. It has no significant overlap with cortisol’s function.
Norepinephrine : Similar to epinephrine but primarily acts as a neurotransmitter in the sympathetic nervous system. It does not have the same metabolic effects on glucose or lipolysis to the same extent as epinephrine.
Growth hormone : Does increase blood glucose and lipolysis , but its primary roles are in growth, protein synthesis , and cell reproduction . Its mechanism and spectrum of action are not as directly parallel to cortisol as epinephrine’s are.
Think about how organs develop as outpouchings or buds from specific sections of the embryonic gut. Rotation plays a big role in how paired structures migrate and fuse — especially around the duodenum.
107 / 109
Category:
Endo – Embryology
The pancreas is formed by the fusion of what two structures?
The pancreas is a vital exocrine and endocrine gland , and its embryological development is a classic example of organ formation through the fusion of two separate structures .
✅ Why “Fusion of ventral and dorsal pancreatic buds” is correct:
The dorsal pancreatic bud arises first from the dorsal aspect of the foregut.
The ventral pancreatic bud arises later near the bile duct .
During embryonic development:
After fusion:
The ventral bud forms the uncinate process and part of the head of the pancreas.
The dorsal bud forms the rest of the head , body , and tail .
This fusion explains the layout of the main pancreatic duct (of Wirsung) and accessory duct (of Santorini) in the mature pancreas.
❌ Why the other options are incorrect:
Fusion of midgut and hindgut : These are large gut segments , not specific pancreatic buds . The pancreas arises from the foregut , not midgut or hindgut.
Fusion of midgut and foregut : Again, irrelevant to pancreatic bud origin. Pancreas is a foregut derivative , and there’s no fusion of gut regions forming the pancreas.
Fusion of two dorsal pancreatic buds : Only one dorsal bud forms. There is no duplication of the dorsal bud in normal development.
Fusion of two ventral pancreatic buds : There is typically only one ventral bud . Sometimes an accessory ventral bud can exist, but it is not the normal embryologic basis for pancreas formation.
Think about which disorder not only alters hormone levels but also causes immune cells to attack soft tissues . Could the eye be a victim in such a systemic autoimmune attack?
108 / 109
Category:
Endo – Pathology
Exophthalmos is most strongly associated with which of the following pathologies?
Exophthalmos , also known as proptosis , refers to the protrusion of the eyeball from the orbit. While multiple conditions may cause it, Graves disease is the most strongly associated due to its unique autoimmune mechanism targeting tissues around the eye.
✅ Why “Graves disease” is correct:
Graves disease is an autoimmune hyperthyroid condition .
The body produces thyroid-stimulating immunoglobulins (TSIs) which stimulate the TSH receptor .
But here’s the key: These antibodies also affect fibroblasts in the orbital tissue , stimulating them to produce glycosaminoglycans (GAGs).
This leads to:
Result: the eye is pushed forward — classic exophthalmos .
This is not just due to hyperthyroidism, but due to immune-mediated infiltration of orbital tissues , making Graves disease the only thyroid disorder with this specific ocular manifestation.
❌ Why the other options are incorrect:
Hypothyroidism : May cause periorbital puffiness due to myxedema, but does not cause true exophthalmos .
Conn syndrome : A condition of primary hyperaldosteronism ; presents with hypertension, hypokalemia , but no eye involvement .
Hypoparathyroidism : Leads to hypocalcemia , causing neuromuscular symptoms (e.g., tetany), not orbital symptoms .
Hyperparathyroidism : Causes hypercalcemia , with symptoms like bone pain , renal stones , and psychiatric disturbances , but not exophthalmos
When an external hormone mimics the function of an internal one, how does the body’s regulatory system respond to avoid excess? Think about feedback loops — are they positive, or negative in this case?
109 / 109
Category:
Endo – Physio
What will be the effect on endogenous cortisol and adrenocorticotropic hormone (ACTH) when exogenous cortisol is given?
🔄 Normal Physiology of the HPA Axis:
Hypothalamus releases CRH (corticotropin-releasing hormone)
CRH stimulates the anterior pituitary to release ACTH
ACTH acts on the adrenal cortex to produce cortisol
Cortisol provides negative feedback to:
The hypothalamus , reducing CRH
The pituitary , reducing ACTH
✅ What happens when exogenous cortisol is administered?
This is a classic example of feedback inhibition — the body’s way of preventing hormone overproduction.
❌ Why the Other Options Are Incorrect:
No change in both : This would suggest no feedback regulation, which is inaccurate for the HPA axis.
Both increase : This contradicts the negative feedback mechanism. Elevated cortisol inhibits ACTH.
Endogenous cortisol increases while ACTH decreases : ACTH does decrease, but endogenous cortisol will also decrease due to less ACTH stimulation.
Endogenous cortisol decreases while ACTH increases : This scenario would occur in primary adrenal insufficiency (like Addison’s disease), not during exogenous cortisol administration .
Your score is
The average score is 0%
Follow us on our Socials ! Thank you.
Restart quiz
Anonymous feedback
Thank you for your feedback.